72
P-1 SOLUT IONS 1 CHAPTER 1 WORKSHEET-1 Ans.1 (a) (i) The highest point- 078065 (.365) (ii) Surveyed tree north of Rampura Khera- 072074. (b) (i) Butri- Left Bank (ii) Padrugarh- Right Bank. (c) (i) 0704- Trellised (ii) 0705- Radial. (d) Presence of seasonal streams, lined perennial well, cart track motorable in the dry season/ dry tanks/ dry river beds. [Any two] (e) The distance between the settlement Bhatana and Makawal along the cart track on the map is 9.8 cms Scale- 2 cm = 1 km Therefore, ½ x 9.8 = 4.9 kms distance on the ground. (f) (i) Dattani from Marol- South West. (ii) Dhavli from Makawal- North East. (g) (i) Monthly fair at Marol. (ii) Agriculture/Farming. (h) (i) The black broken line in 0807- Disappearing drainage pattern. (ii) The vertical black line close to Easting 10- Longitude line- 72°35’ E. (i) Presence of Lined perennial wells all around the settlement. (j) (i) 0905- Broken ground, seasonal streams. (ii) 1003- PO, Temple, cart track, seasonal tank, Lined perennial well. [Any one] Ans.2 (a) (i) Spot height .542-977827 (ii) Temple at Juvol- 926828 (b) (i) R.F stands for Representative Fraction. It is the ratio between the distance on the map to the distance on the corresponding ground and is in fraction. The numerator denotes the length on the map and the denominator denotes the actual distance on the ground. E.g. 1:50,000. (ii) Eastings 91 to 01 = 10 grids and Northings 73 to 83 = 10 grids Each grid = 1 sq.km (scale given 2 cm = 1 km) 10 x 10 = 100 sq. kms. (c) The distance between Juvol and Arniwada on the map is 5 cms Scale given 2 cm = 1 km ½ x 5= 2.5 kms distance between the Juvol and Arniwada on the ground. (d) (i) East to West. (ii) Right Bank. (e) (i) Well irrigation is the main form of irrigation. (ii) 1) Since this area in G.S. 9478 is a sandy region, the water is absorbed in the sand. 2) Due to high temperature the water evaporates and do not join the river. [Any one] (f) (i) Ridge/ Col. (ii) 4R represents the relative depth of the lined perennial well. (g) (i) 9576 -Nucleated or Compact. 9774 - Scattered or Dispersed. 9574- Radial drainage pattern. 9382- Trellis drainage pattern. INTERPRETATION OF TOPO-GRAPHICAL MAPS

CHAPTER INTERPRETATION OF TOPO-GRAPHICAL MAPS

  • Upload
    others

  • View
    4

  • Download
    0

Embed Size (px)

Citation preview

Page 1: CHAPTER INTERPRETATION OF TOPO-GRAPHICAL MAPS

P-1S O L U T I O N S

DRAMA

1CHAPTER

1 WORKSHEET-1Ans.1 (a) (i) The highest point- 078065 (.365)

(ii) Surveyed tree north of Rampura Khera- 072074. (b) (i) Butri- Left Bank

(ii) Padrugarh- Right Bank. (c) (i) 0704- Trellised

(ii) 0705- Radial. (d) Presence of seasonal streams, lined perennial well, cart track motorable in the dry season/ dry tanks/

dry river beds. [Any two] (e) The distance between the settlement Bhatana and Makawal along the cart track on the map is

9.8 cms Scale- 2 cm = 1 km Therefore, ½ x 9.8 = 4.9 kms distance on the ground.

(f) (i) Dattani from Marol- South West. (ii) Dhavli from Makawal- North East.

(g) (i) Monthly fair at Marol. (ii) Agriculture/Farming.

(h) (i) The black broken line in 0807- Disappearing drainage pattern. (ii) The vertical black line close to Easting 10- Longitude line- 72°35’ E. (i) Presence of Lined perennial wells all around the settlement. (j) (i) 0905- Broken ground, seasonal streams.

(ii) 1003- PO, Temple, cart track, seasonal tank, Lined perennial well. [Any one] Ans.2 (a) (i) Spot height .542-977827

(ii) Temple at Juvol- 926828 (b) (i) R.F stands for Representative Fraction. It is the ratio between the distance on the map to the

distance on the corresponding ground and is in fraction. The numerator denotes the length on the map and the denominator denotes the actual distance on the ground. E.g. 1:50,000.

(ii) Eastings 91 to 01 = 10 grids and Northings 73 to 83 = 10 grids Each grid = 1 sq.km (scale given 2 cm = 1 km) 10 x 10 = 100 sq. kms. (c) The distance between Juvol and Arniwada on the map is 5 cms Scale given 2 cm = 1 km ½ x 5= 2.5 kms distance between the Juvol and Arniwada on the ground. (d) (i) East to West. (ii) Right Bank. (e) (i) Well irrigation is the main form of irrigation. (ii) 1) Since this area in G.S. 9478 is a sandy region, the water is absorbed in the sand. 2) Due to high temperature the water evaporates and do not join the river. [Any one] (f) (i) Ridge/ Col. (ii) 4R represents the relative depth of the lined perennial well. (g) (i) 9576 -Nucleated or Compact. 9774 - Scattered or Dispersed. 9574- Radial drainage pattern. 9382- Trellis drainage pattern.

INTERPRETATION OF TOPO-GRAPHICAL MAPS

Page 2: CHAPTER INTERPRETATION OF TOPO-GRAPHICAL MAPS

P-2 G E O G R A P H Y - X

(h) (i) North East. (ii) Chitrasani is well connected by metalled road and railway and has good public utility facilities

like PO, RS. (i) (i) Man made feature- Temple and Lined Perennial Well. Natural feature- Ridge/Col. (ii) Broken Ground. (j) (i) Cart Track and Metalled Road. (ii) Agriculture due to the presence of large number of Lined Perennial Wells.

Ans.3 (a) (i) Surveyed tree 219 north east of Pirojpura settlement- 979756. (ii) ∆364inthesouthernpartofthemapextract-957744.

(b) Direction of flow of Banasriver is from north east to west. Evidence is decreasing spot height from 233 in 9681 to 177 in 9380.

(c) (i) It is the relative height of sand dune which is 12 m. (ii) It is contour line showing 180 m. above mean sea level.

(d) Area of the region between 93 and 99 eastings and 76 and 81 northings is 30 km² (Area of one square is 1 km²). There are thirty squares within this region.

(e) (i) North West. (ii) 542-198=344 mt.

(f) (i) It is a perennial water channel in the seasonal stream. (ii) It is sand dune.

(g) (i) 9782- Radial drainage pattern. (ii) 9478- Disappearing drainage pattern.

(h) The region has hot and dry climate with seasonal scanty rainfall. Evidence is presence of seasonal streams, lined perennial well, broken ground, etc.

(i) Manmade feature- cart track, temple, lined perennial, permanent hut. Natural feature- broken ground, seasonal river.

(j) (i) Black vertical line is the line of longitude. (ii) RS denotes railway station.

[ICSE Marking Scheme 2016]

ll

WORKSHEET-2Ans.1 (a) (i) Hamirpura- 0123.

(ii) Temple – 078187. (b) (i) Perennial lined well. (ii) Towards Northwest. (c) Difference between: (i) Settlements in 0725- Scattered or Dispersed. Settlement Idarla- Compact or Nucleated. (ii) Drainage pattern in 0624- Trellised. Drainage pattern in 0824- Dendritic. (d) 300 m above mean sea level is the value of the contour line in grid square 0226 Contour Interval- 20 m. (e) (i) Occupation is agriculture. (ii) Lined perennial well is the main source of water. (iii) Mode of transport is cart track. [Any two points] (f) (i) Broken ground is formed due to the result of weathering caused by alternate wet and dry

periods which means that the rainfall is seasonal. (ii) The cart track which is motorable only during the dry season.

Page 3: CHAPTER INTERPRETATION OF TOPO-GRAPHICAL MAPS

P-3S O L U T I O N S

(g) (i) Two natural features in 0527- Seasonal streams, knoll, barren ground. (ii) Two manmade features in 0325- Cart track, permanent hut, cultivated land. (h) Dantrai is the most important settlement because of the presence of Post Office, police chowki, high

density of population due to large number of settlements. (i) On the south of Northing 21, there 30 grids. Each grid is 1 square kilometres, therefore, total area of

the region is 30 km² or square kilometres. (j) (i) The black vertical line is the Line of Longitude. (ii) 302 is the spot height (.302 m) above mean sea level.

Ans.2 (a) 1511, 1711, 1811, 1903. Dry tank with embankment. (b) 1606- Radial pattern.

1608- Trellis. (c) (i) Gautam Maharishi Mandir- 200071.

(ii) .443- 179052. (d) Vajna (1503) Two reasons: Presence of a police chowki, Metalled road (e) (i) West.

(ii) Souh West. (f) (i) Causeway- A raised road over a small stream but not a bridge/ A raised metalled road over a

non-perennial stream or marshy area or which serves as a temporary bridge. (ii) Falls 25 m- It indicates a waterfall which is falling from a height of 25 metres.

(g) (i) 1 Hour. (ii) 15 sq. kms.

(h) (i) 1610- Seasonal stream, broken ground, dry stream. (ii) 1903- Permanent settlement, embankment.

(i) Presence of broken ground, causeways, motorable in dry season. (j) Highest point is 1409 m and contour height is 2402 is 80 m. Difference is 1409-80= 1129 metres. [ICSE Marking Scheme 2015]

ll

WORKSHEET-3Ans.1 (a) (i) ∆217- 940793.

(ii) Lined well near Chekhla- 925815. (b) (i) Balram Nadi. (ii) Towards North West. (c) (i) Broken grounds. (ii) Trellised. (d) (i) 9573/9680/0076. (ii) 9978. (e) 9782- Conical Hill. 9574- Ridge. (f) Due to scanty rainfall cultivation is limited in the map extract. Two reasons- (i) The northern part of the map is a highland. (ii) The southern part is covered with sand dunes. (g) Antroli from Sangla- North East. Chekhla from Sangla- North.

Page 4: CHAPTER INTERPRETATION OF TOPO-GRAPHICAL MAPS

P-4 G E O G R A P H Y - X

(h) Seasonal rainfall. Presence of broken grounds, open scrub, sand dunes, water channel. [Any one] (i) The distance between Chitrasani and Pirojpura on the map is 4.6 cms. Scale given- 2 cm = 1 km Therefore, ½ x 4.6= 2.3 kms on the ground. (j) (i) Scattered. (ii) Southern part of the map.

Ans.2 (a) Malgaon in G.S. 1520. (b) Spot height 270 in 145132. (c) (i) South East. (ii) Cart track and Metalled road. (d) (i) 280- Contour height 280m. (ii) 281- Spot height 281m. (e) In Bamba settlements are nucleated and temporary. In square 1813 the settlements are dispersed and permanent. (f) Four facilities in Anadra- PTO, police chowki, DB (Dak Bunglow), dispensary. (g) Telephone. (h) (i) Brackish in 1915- It means that water drinking and agricultural purposes. (ii) Causeway in 1715- A raised road over a small stream but not a bridge/ A raised metalled road

over a non-perennial stream or marshy area or which serves as a temporary bridge. (iii) 6r in 1618- A relative depth of the bank of the river is 6 m. (i) (i) Sipu Nadi is in its middle course as the region is a flat level land, there are meanders and the

river is sluggish. (ii) The rainfall is seasonal because there are broken grounds, barren patches of land, motorable in

dry season and causeway. (j) The distance in kilometres between the distance stone 20 in 1818 and the causeway in 1715 along the

metalled road on the map = 7 cms. Scale given- 2 cm = 1 km ½ x 7= 3.5 kms is the distance on the ground.

ll

WORKSHEET-4Ans.1 (a) (i) The brick kiln near village Serua- 088131

(ii) The temple near Asav- 059128 (b) 0916- Radial. 0712- Trellis. (c) (i) Stony waste- 1014. (ii) Open scrub- 0816. (d) The distance between the causeways in grid square 0512 and 0808 on the map is 11.4 cms. Scale – 2 cm = 1 km. Therefore, ½ x 11.4 = 5.7 kms is the distance along the metalled road. (e) (i) Broken ground. (ii) It is formed due to the result of weathering caused by alternate wet and dry periods which

means that the rainfall is seasonal. (f) (i) Nucleated pattern of settlement. (ii) Settlements are closely located. (g) General direction of Sipu Nadi is South West. The spot height is decreasing from .261 (1116) to .237

(0408). (h) In G.S. 0313, the probable occupation is government service in PTO(Post and Telegraph Office) and

Police chauki and in 0413, the occupation is agriculture. (i) (i) Cart track connects Marol and Mitan.

Page 5: CHAPTER INTERPRETATION OF TOPO-GRAPHICAL MAPS

P-5S O L U T I O N S

(ii) Metalled road connects Revdar and Karaunti. (j) The area receives seasonal rainfall, it is evident due to the presence of broken ground, stony waste,

dry streams, open scrubs, causeway. Ans.2 (a) (i) Triangulated height 307- 859843.

(ii) The spot height 196- 835916. (b) (i) The confluence of the Sipu River and Mahadeviyo Nala- 8189. (ii) Sheet rock- 8088/8188. (c) The shortest distance between the temple in grid square 8192 and the perennial lined well at

Bhakodar 8188 on the map is 7 cm. Scale- 2 cm = 1 km Therefore, ½ x 7 = 3.5 kms is the shortest distance on the ground. (d) (i) Black curved line in 7788- Broken grounds. (ii) The blue line in the bed of the Sipu River- Seasonal stream with water channel. (e) (i) The general pattern of settlement is compact or nucleated. (ii) Settlements are closely located. (f) The chief form of irrigation is the Lined perennial well. It is necessary because the river and the

rainfall are seasonal. (g) (i) Chief mode of transport is Cart track. (ii) A large number of red lines are crisscrossing the entire map. (h) (i) North-east. (ii) South-west. (i) (i) Trellis. (ii) 25r is the relative depth of the lined perennial well which is 25 metres. (j) (i) R.F. stands for Representative Fraction. It is the ratio between the distance on the map to the

distance on the corresponding ground and is in fraction. The numerator denotes the length on the map and the denominator denotes the actual distance on the ground. E.g. 1:50,000.

(ii) RF is 1 : 50,000.ll

WORKSHEET-5Ans.1 (a) (i) Spot height 261- 1107.

(ii) Post office- 1003. (b) 1606 - Radial. 1007- Trellised. (c) Two natural features in G.S. 0910- Broken grounds and Seasonal dry stream, Dry tank. (d) The distance from the causeway East of Was to the distance stone 10 on the map is 16 cms. Scale- 2 cm = 1 km Therefore, ½ x 16= 8 kms is the distance on the ground. (e) (i) Universally accepted scale is Representative Fraction (R.F.) (ii) The length of the given map is 10 km (Total 10 grids and each side of grid is 1 km, as per scale 2

cm = 1 km) (f) (i) A temple south of Dhavli settlement- 111073. (ii) Surveyed tree 277- 177056. (g) Cart track and metalled road. (h) Agriculture and Forestry. (i) Gentle slopes are shown by spacing the contour lines far apart while steep slopes are shown by the

closely placed contour lines. (j) (i) 3r in 1103- Relative depth of the dry tank. (ii) Open scrub in 1502- It means semi-xerophytic vegetation which grows in seasonal rainfall areas

and is not a cultivable land. It is generally used for cattle rearing.

Page 6: CHAPTER INTERPRETATION OF TOPO-GRAPHICAL MAPS

P-6 G E O G R A P H Y - X

(k) Dattani from Chandela- West of Chandela. Dhavli from Chandela- North-West of Chandela (l) Two man-made features in grid square 1210- Permanent hut / settlements and Lined perennial well,

cart track. (m) Motorable in dry season means that the cart track can be used only in dry season i.e. from September

to May as during rainfall it is flooded with water. It indicates that this region receives scanty and seasonal rainfall.

Ans.2 (a) Sunset Point from Anadra- South-East. (b) Drainage Pattern in G.S. 2315- Radial. (c) Broken ground, Seasonal streams, motorable in dry season, dry tank. (d) The distance along the metalled road from the causeway in grid square 1715 to the distance stone

marked 20 in grid square 1818 on the map is 6.2 cm. Scale- 2 cm = 1 km ½ x 6.2 = 3.1 km is the distance on the ground. (e) Representative Fraction is only a fraction and is independent of any particular unit of measurement

and has a universal application. (f) (i) Anjini Devi ka Mandir- 229160. (ii)∆1327-217106. (g) Three different kinds of road in G.S. 2411 are Metalled Road, Pack track, Gravelled or unmetalled

road while in G.S. 2515 it is Footpath. (h) The two occupations are- Agriculture as there is yellow coloured region and Forestry as the entire eastern region is green coloured. (i) The slope in G.S. 2115 is Gentle slope because the contour lines are largely spaced and far apart and

in G.S. 1811, it is Steep slope because the contour lines are closely drawn to each other. (j) (i) The fire line in grid square 2316- To prevent the spread of fire in the jungles. (ii) The pipeline in grid square 2209- To supply water from the reservoir to Abu. (k) (i) The red square in grid square 2514- Temporary Huts. (ii) 4r in grid square 1612- Relative height of an embankment which is 4 m from the surrounding

surface. (l) Three features seen which attract holiday makers to Abu- (i) The Dilwara temples, Jaivilas Palace. (ii) The two lakes- Nakhi Talao and Alwar Talao. (iii) Sunset Point.

ll

WORKSHEET-6Ans.1 (a) (i) ∆364-957744.

(ii) The temple at Rampura- 954805. (b) Two features that show the region has seasonal rainfall: (i) Presence of Broken ground and seasonal streams. (ii) Presence of sand dunes. (c) The distance between Juvol (923826) and Arniwada (944817) on the map is 4.8 cm Scale -2 cm = 1 km Therefore, ½ x 4.8=2.4 km is the distance on the ground. (d) Lined perennial well is the chief form of irrigation. A large number of blue dots can be seen in the

map extract. (e) (i) Since this area in G.S. 9478 is a sandy region, the water is absorbed in the sand. (ii) Due to high temperature the water evaporates and do not join the river. (iii) This region receives seasonal or low rainfall. [Any two points]

Page 7: CHAPTER INTERPRETATION OF TOPO-GRAPHICAL MAPS

P-7S O L U T I O N S

(f) (i) Black broken lines in 9575- disappearing stream. (ii) Black curved lines in 9879- Broken ground (g) Two landforms shown by the contours in grid square 9876- Ridge and Watershed. (h) (i) The general direction of Balaram Nadi is from South-East to North-West. (ii) Balaram Nadi joins on the left bank of the main river. (i) Agriculture is the main occupation of the people in this region as the vast tract of yellow colour

shows cultivable land and the presence of large number of lined perennial wells. (j) Scale of the map is the proportion between the distance on the map and the corresponding distance

on the ground. The scale of the map provided is 2 cm to 1 km.

ll

WORKSHEET-7Ans.1 (a) (i) The temple in village Dhavli- 111073.

∆480- 137045. (b) (i) 6r in grid square 1903- Relative height of embankment which is 6 meters. (ii) The word Brackish in grid square 1403- It means well containing salty water which is unfit for

drinking and agriculture. (c) (i) It represents a cliff. (ii) Red dotted lines are footpath which is the only means of transport in the mountainous

region. (d) The region receives seasonal rainfall. The reasons are the presence of large number of seasonal

streams, broken ground, Open scrub. (e) Drainage pattern in G.S. 1606 is Radial and in G.S. 1708 is Trellised. (f) (i) Pattern of settlement in G.S. 1904- Scattered or Dispersed. (ii) Dattani has a Post office. (g) Ridge and Saddle. (h) The ground distance between the causeway in 1502 and the distance stone marked 14 in grid square

1203 is 7.8 cm Scale- 2 cm = 1 km Therefore, ½ x 7.8= 3.9 km is the distance on the ground. (i) (i) It is a hilly terrain. (ii) It is covered with thick forests. (j) (i) Highest spot height-1023 Contour height - 260 Difference in height- 763 m. (ii) North-West.

Ans.2 (a) (i) The brown line in grid square 1516 is called Contours. (ii) The figure written along this line indicates the height in meters of the contour line, i.e. 280 which

is above mean sea level. (b) Cart track and Metalled road. (c) Gulabganj is on the left bank of Sipu Nadi. It gets its water supply from the Lined perennial well. (d) Fire line is the space that is kept between the trees approximately, 5 meters apart to prevent forest

fire. It is necessary to avoid the spreading of fire. (e) Nature of the canal- Perennial The length of the canal on the map is 10.6 cm Scale- 2 cm = 1 km, therefore, ½ x 10.6 = 5.3 km. (f) 2118- Radial. 2114- Dendritic.

Page 8: CHAPTER INTERPRETATION OF TOPO-GRAPHICAL MAPS

P-8 G E O G R A P H Y - X

(g) Anadra is the most important because it has facilities of PTO (Post and Telegraph Office), DB (Dak Bunglow), Police chauki, Dispensary and is connected with metalled road.

(h) (i) North-East. (ii) Gulabganj forms a nodal centre which means it is connected with all other villages whereas

Udwariya is not so. ll

WORKSHEET-8Ans.1 (a) (i) ∆592-928890.

(ii) Temple- 912883. (b) (i) South-West. (ii) The blue line in Arado N indicate Water Channel. (c) The distance between Velavas (868895) and Ranol (883888) on the map is 5 cm. Scale -2 cm = 1 km Therefore, ½ x 5= 2.5 km is the distance on the ground. (d) (i) 7r indicates Relative depth of the Lined perennial well. (ii) Lined perennial wells. (e) Seasonal rainfall. Presence of broken ground, seasonal dry streams, water channel, Island in the river bed, open scrub,

jungle [Any one point] (f) Contour lines. These are the imaginary lines joining places of same height above the sea level. (g) Two types of vegetation- Open mixed jungle in G.S. 9187 and Dense mixed jungle in G.S. 9385. (h) (i) Conical Hill. (ii) Island. (i) (i) Human settlement in 8989- Linear settlement. (ii) Drainage in grid square 9185- Radial. (j) Agriculture is the general occupation of people. Presence of Lined perennial wells and yellow-wash.

Ans.2 (a) (i) Dadarla- 1020. (ii) Bhamra- 0420. (b) Dantrai is the largest settlement. PO (Post office) is one communication facility. (c) R.F. stands for Representative Fraction. It is the ratio between the distances on the map to the

distance on the corresponding ground and is in fraction. The numerator denotes the length on the map and the denominator denotes the actual distance on the ground. E.g. 1:50,000

R.F. = Distance on the map (MD

Distance on the ground (GD) R.F. of the map is 1:50,000 i.e. 1 unit on the map is equal to 50,000 units on the ground. (d) Total grids from Eastings 02-10 (West to East) is 9 Total grids from Northings 17-28 (South to North) is 11 Each side of grid is 2 cm, Scale- 2 cm = 1 km From Eastings 02-10 (West to East) – 18 cm (9×2) = 18 ÷ 2= 9 km (L) From Northings 17-28 (South to North)- 22 cm (11 × 2) = 22 ÷ 2 = 11 km (B) Area = L×B Therefore, 9 x 11 = 99 sq.km. (e) Contour Interval- is the vertical difference between two consecutive contour lines which remains

constant throughout the map, e.g. 20 metres. Contour Interval of the map is 20 meters. (f) Highest spot height is 339 and the lowest spot height is 263. They are in metres.

Page 9: CHAPTER INTERPRETATION OF TOPO-GRAPHICAL MAPS

P-9S O L U T I O N S

(g) (i) Spot height 339- 042276. (ii) Permanent hut located near Dhad Talao- 077225. (h) (i) Main occupation – Agriculture. (ii) Religion- Hindu. (i) Natural source of irrigation- Sukli Nadi and seasonal streams. Man made source of irrigation- Lined perennial well. (j) Mode of transportation- Cart track. Special feature of this mode is Motorable in dry season.

ll

Page 10: CHAPTER INTERPRETATION OF TOPO-GRAPHICAL MAPS
Page 11: CHAPTER INTERPRETATION OF TOPO-GRAPHICAL MAPS

P-11S O L U T I O N S

DRAMA

1CHAPTER

2 WORKSHEET-9Ans.1

(d) Red Soil

(b) Chenab

(f) Coromandel Coast

(a) Western Ghats

(a) Eastern Ghats

(c)SW Monsoon

Winds ofBay of Bengal

Branch

(h) Singhbhum (Iron)

(h) Jharia (Coal)(j) Allahabad

(e) 82½ ˚E

(g)Nathu-la Pass

(i)Bihar

(Highest densityof population)

Map not to scale

LOCATION, EXTENT AND PHYSICAL FEATURES

Page 12: CHAPTER INTERPRETATION OF TOPO-GRAPHICAL MAPS

P-12 G E O G R A P H Y - X

Ans.2

(a) Tropic of Cancer

(b)Godavari

(i)BlackSoil

(j)

(Denselypopulated

state inSouth India)

(g)Coromandel Coastreceives rainfall in

Oct-Nov

(f)Gulf ofKutch

(h)Delhi

(c) Garo Hills

(d)

(e)NE Monsoon Winds

C

Kerala

Map not to scale

Page 13: CHAPTER INTERPRETATION OF TOPO-GRAPHICAL MAPS

P-13S O L U T I O N S

Ans.3

(a) 82½˚E

(f)Western Rajasthan

(h)Kolkata

(e)Vindhayas

(b)Mahanadi (c)

Chilka Lake(j)

MumbaiHigh

(g)Alluvial

Soil

(d) Gulf of Mannar

(i)SW Monsoon

Winds

Sparselypopulated

Map not to scale

[ICSE Marking Scheme, 2016]

ll

Page 14: CHAPTER INTERPRETATION OF TOPO-GRAPHICAL MAPS

P-14 G E O G R A P H Y - X

WORKSHEET-10

Ans.1

(c)Malabar

Coast

(a)Nilgiris

(i)Laterite

Soil

(f) Allahabad(b) Malwa

(d) Gomti

(h) Digboi

(j) Karakoram Pass

(g)Western

Disturbancesor Westerly Winds

(e)Andaman

SeaMap not to scale

[ICSE Marking Scheme, 2015]

Page 15: CHAPTER INTERPRETATION OF TOPO-GRAPHICAL MAPS

P-15S O L U T I O N S

Ans.2

(e)Karakoram

Range

(a) 82½˚E (Standard Meridian)

(b) Yamuna

(d)Nathu-la Pass

(f)Arunachal PradeshSparsely Populated

(h) Vishakhapatnam

(c)Gulf of

Khambhat

(j)(S)

(g)Laterite Soil (i)

SW MonsoonWinds

Map not to scale

[ICSE Marking Scheme, 2014]

Page 16: CHAPTER INTERPRETATION OF TOPO-GRAPHICAL MAPS

P-16 G E O G R A P H Y - X

Ans.3

(c) Chota Nagpur Plateau

(i) Jharia

(f)Red Soil

(d)Gulf ofKutch

(g)South West

Monsoon Winds(a) Chennai

(e) Indo-Gangetic Plains

(h) Satpura

(b)Godavari

(j) Northern Circars

Map not to scale

ll

Page 17: CHAPTER INTERPRETATION OF TOPO-GRAPHICAL MAPS

P-17S O L U T I O N S

WORKSHEET-11

Ans.1

(a) Tropic of Cancer 23½ ˚N

(e) Karakoram Pass

(h) Srinagar

(d) Aravalli

(j)

(O)

(g)BlackSoil

(i)N.E. Trade Winds(f)

Kerala(Densely

Populated)

(b) Narmada

(c) Chilka Lake

Map not to scale

Page 18: CHAPTER INTERPRETATION OF TOPO-GRAPHICAL MAPS

P-18 G E O G R A P H Y - X

Ans.2

(i) Jharia

(b) Krishna

(d)KonkanCoast

(h)Western Ghats

(j)Western Uttar Pradesh

(f)Alluvial Soil

(a) Kochi

(c)MalwaPalteau

(e) Gulf of Mannar

(g)SW Monsoon

Winds

Densely Populated Area

Map not to scale

Page 19: CHAPTER INTERPRETATION OF TOPO-GRAPHICAL MAPS

P-19S O L U T I O N S

Ans.3

(j)SW Monsoon

(f) Hyderabad(d) Eastern Ghats

(h) Palk Strait

(a) Chenab

(b) Chambal

(i) Assam(Tea growing region)

(c) Nilgiri Hills

(g) Sunderbans

Winds

Map not to scale

Page 20: CHAPTER INTERPRETATION OF TOPO-GRAPHICAL MAPS

P-20 G E O G R A P H Y - X

Ans.4

(i) Tropic of Cancer

(b) Godavari(h)Offshoreoilfield

(c) Aravalli

(a) Satluj

(g)Laterite

Soil

(f) Bangaluru

Map not to scale

(j) Bay ofBengalcurrent

ll

Page 21: CHAPTER INTERPRETATION OF TOPO-GRAPHICAL MAPS

P-21S O L U T I O N S

WORKSHEET-12

Ans.1

(h) Laterite Soil

(f)N.E. Monsoon Winds

(d) Ravi

(g) Malabar Coast

(e) Khasi

(a) Satpura

(b) Krishna

Map not to scale

Page 22: CHAPTER INTERPRETATION OF TOPO-GRAPHICAL MAPS

P-22 G E O G R A P H Y - X

Ans.2

(e)Gulf ofMannar

(f) Brahmaputra

(c) Indus

(a)Mt. Everest

(b) 82½˚E Longitude

(d)KathiawarPeninsula

Map not to scale

Page 23: CHAPTER INTERPRETATION OF TOPO-GRAPHICAL MAPS

P-23S O L U T I O N S

Ans.3

a (ii)Western

Ghats

c (i)Red Soil

d (ii) Yamuna

c (ii) Thar Desert

c (iii)Region of

Winter Rainfall

Map not to scale

Page 24: CHAPTER INTERPRETATION OF TOPO-GRAPHICAL MAPS

P-24 G E O G R A P H Y - X

Ans.4

a (i) Karakoram Range

c (iii) Godavari

a (ii) Eastern Ghats

a (iii) Nilgiris

d (ii) Direction of Summer Monsoon

b (ii)BlackSoil

T

d (i) 1. Arunachal Pradesh sparsely populated

Map not to scale

ll

Page 25: CHAPTER INTERPRETATION OF TOPO-GRAPHICAL MAPS

P-25S O L U T I O N S

DRAMA

1CHAPTER

3 WORKSHEET-1Ans.1 Retreating North –East Monsoon winds. Ans.2 (i) Since Agra is situated in the interior part of the country and is experiencing the Continental type

of climate and Mumbai is a coastal city experiencing equable climate therefore annual range of temperature at Agra is more.

(ii) North West part of India like Punjab, Delhi, Uttarakhand. Ans.3 (i) The Aravalli Hills lie parallel to the South West Monsoon winds coming from the Arabian Sea and thus

cannot shed any moisture in Western Rajasthan.(ii) The temperature of the region is so high in summers that the monsoon winds get dry and do not shed

any rain there.

Ans.4 (i) Tropical monsoon climate.(ii) Effect of monsoon wind / Latitudinal extent / Tropic of cancer passing through its centre / effect of

Himalayas. [ICSE Marking Scheme, 2016]

Ans.5 (i) South West monsoon winds from the Bay of Bengal branch.(ii) Mango Showers or Cherry Blossoms. [ICSE Marking Scheme, 2016]

Ans.6 Rainfall in Rainfall in June – Sep Dec – Feb Orographic Cyclonic Heavy/torrential Light Causes destruction Beneficial

[ICSE Marking Scheme, 2015] [Any two] Ans.7 (i) Station A

Annual Range of temperature= Maximum temperature – Minimum temperature 32.5 – 24.5= 8°C(ii) 183.2 cms.(iii) Station A is situated on the eastern coast. Retreating North-East winds bring rainfall to this region.

Ans.8 (i) Kanpur lies in the interior, far from moderating effect of the sea.

(ii) Kochi is warmer because it is closer to the Equator than Mumbai.

(iii) It is because the Monsoon winds come to India from the south west and so reach the west coast before it can reach the Ganga plain. [ICSE Marking Scheme, 2016]

Ans.9 (i) Annual rainfall- 128.7 cm.(ii) 32.5° C – 24.5° C = 8°C(iii) East Coast. It gets most of the rain in the season of retreating monsoon.

[ICSE Marking Scheme, 2016]

Ans.10 (i) Hottest month- May.(ii) Annual rainfall- 131.9 cm.(iii) Retreating Monsoon.

Ans.11 (i) 36.2° C – 24.6° C= 11.6° C(ii) Annual Rainfall- 140.9 cms.(iii) The station is located on the east coast because it gets most of its rain in the winter months.

Ans.12 (i) Annual Rainfall- 114.3 cms.(ii) Chennai, because it is receiving most of its rainfall from October to December.(iii) Retreating Monsoon Season.

Ans.13 (i) Mean Annual temperature- 28°C

CLIMATE

Page 26: CHAPTER INTERPRETATION OF TOPO-GRAPHICAL MAPS

P-26 G E O G R A P H Y - X

(ii) Total rainfall- 180.5 cms.(iii) The station has a maritime climate as the range of temperature is very low i.e. 6.6°C.

Ans.14 (i) It is due to the effect of land and sea breeze caused by the differential rate of heating and cooling of the land and sea which results in an equable and moderate climatic conditions.

(ii) The Aravalli hills lie parallel to the south west monsoon winds of the Arabian Sea branch and do not hinder the winds. Thus, Rajasthan remains dry. On the other hand, the winds blowing from the Bay of Bengal branch sheds its moisture on its way and gets dry by the time it reaches Rajasthan. Rajasthan also lies on the leeward side of the Aravalli Hills and thus receives less than 25 cm of rainfall.

(iii) The Coromandel Coast, situated on the eastern coast, receives rain in winter due to the cyclonic depressions that prevail over the Bay of Bengal due to low pressure conditions. It doesn’t receive rain in summer as it lies in the leeward side of the Arabian Sea branch.

ll

WORKSHEET-14Ans.1 Jaipur is situated in the interior part of the country, far away from the influence of the sea while

Mumbai is a coastal city and is influenced by sea. Thus, Jaipur has a higher annual range of temperature than Mumbai.

Ans.2 (i) Kerala- Mango showers are good for the tea and coffee crop.(ii) West Bengal- Nor’wester is good for the rice and jute crop.

[ICSE Marking Scheme, 2014]

Ans.3 Punjab - rain from the westerly depressions/western disturbances. Tamil Nadu - rain from the north east monsoons. [ICSE Marking Scheme, 2014]

Ans.4 Mountains or relief is responsible for the distribution of rainfall in India. For example, the Western Ghats act as a barrier for the south west monsoon winds which results in heavy rainfall on the windward side of the Ghats while the leeward side of the Western Ghats remains dry.

Ans.5 Mumbai lies on the windward side of the Western Ghats and receives rainfall from the south-west monsoon in the summer season while Chennai is located on the eastern side and receives rainfall from the retreating north-east monsoon winds which picks up moisture from the Bay of Bengal and shed rainfall there.

Ans.6 (i) The driest month is March with the least rainfall of 1.3 cms. (ii) The annual rainfall is 129.2 cms. (iii) The annual range of temperature is 32.5 – 20.4 = 12.1°CAns.7 (i) The North East monsoon winds blow over most part of India from land to the sea and thus, don’t

bring any rain to most of India.(ii) The Mango Showers are beneficial for the growth of mango, tea and coffee.(iii) The latitudinal extent of India is between 8° N to 37° N and the Tropic of Cancer passes through the

centre of India. The southern part of India lies in the tropical zone and experiences high temperature throughout the year while the northern part lies in the temperate zone and experiences extremity of temperatures i.e. too cold in winters and too hot in summers.

Ans.8 (i) Annual Range of temperature of Station B is 30.0°C – 24.4°C= 5.6°C.(ii) Annual rainfall of Station A is 67.4 cms.(iii) Station B since it receives heavy amount of rainfall in the month of June- July, i.e. 50.2 cms and 61.0

cms respectively. Ans.9 (i) The annual rainfall experienced by the station is 129.2 cms.

(ii) Annual range of temperature is –32.5°C – 20.4°C = 12.1°C(iii) November is the wettest month.

Ans.10 (i) Annual range of temperature of Station A is 30.0°C – 24.4°C = 5.6°C. The range is a small one because it is a coastal city.

(ii) Station B has the lower temperature because it is situated at a high altitude of about 800 m above the sea level.

(iii) Annual rainfall of Station B is 31.8 cms. ll

Page 27: CHAPTER INTERPRETATION OF TOPO-GRAPHICAL MAPS

P-27S O L U T I O N S

WORKSHEET-15Ans.1 Western coastal plains receive more rainfall because the Western Ghats check the south-west moisture

laden winds to cause heavy rainfall. Ans.2 The retreating North-East winds which picks up moisture from the Bay of Bengal and shed rainfall in

Tamil Nadu. This rain is beneficial for growing rice and coffee. Ans.3 The Northern Plains of India experiences continental type of climate. Its main characteristic is that it

is too hot in summers and too cold in winters and has a high range of temperature. Ans.4 Mangalore lies on the windward side of the Western Ghats on the west coast of India while Mysore

lies on the leeward side of the Eastern Ghats. During summer, the south west monsoon winds strikes the Western Ghats and shed good amount of rainfall whereas the east coast receives rain from the north-east monsoon winds that picks up moisture from the Bay of Bengal.

Ans.5 The North West part of India, i.e. Punjab, Haryana, Delhi, Uttarakhand, western Uttar Pradesh etc. receives rain from Western Disturbances. This rainfall is beneficial for the cultivation of Rabi crops i.e. wheat and barley.

Ans.6 (i) Annual range of temperature is 33.5°C – 13.7°C = 19.8°C(ii) Total rainfall is 67.4 cms.(iii) November is the driest month.

Ans.7 (i) Annual rainfall of Station is 67.0 cms.(ii) Annual range of temperature in Station B is 33.0°C – 24.4°C = 5.6°C(iii) Station B has an equable climate.

Ans.8 (i) Annual rainfall of Station A is 68.4 cms.(ii) Annual Range of Temperature of Station B is 33.1°C -23.1°C = 10°C(iii) The North East monsoon winds bring most of the rainfall to Station B. From the month of October

to December, the winds while crossing the Bay of Bengal picks up moisture from there, enters the eastern coasts and strikes the Eastern Ghats and sheds rainfall.

Ans.9 (i) Average Annual Rainfall of Station B is 128.7 ÷ 12 = 10.72 cms. (Total Annual Rainfall ÷Total no.of months) (ii) Station A is located away from the sea because the range of temperature is high.(iii) Annual Range of Temperature for Station A is 33.9°C – 14.9°C = 19°C.(iv) December is the driest month in Station A.

Ans.10 (i) Annual Rainfall for Station A is 67 cms.(ii) Annual Range of temperature at Station B is 30.0°C – 24.4°C = 5.6°C(iii) Station A lies in the Northern Hemisphere.(iv) Station B has an equable climate.

ll

WORKSHEET-16Ans.1 The factors that influence the climate of India are-

(a) Latitude : the further one moves away from the equator, the colder it gets as the rays of the sun get more oblique. Hence places in the south like Kochi, Chennai, Bangalore and others are warmer than places in the north like Delhi, Agra, Srinagar and others due to proximity to the equator.

(b) Altitude : the higher one goes above sea-level, the colder it gets as the atmosphere is heated from below. Hence places at a higher altitude like Simla, Mussoorie, Panchgani and others are cooler than places at sea level like Mumbai, Bhopal, Jaipur and others.

(c) Distance from the sea : The further one moves away from the sea, the higher will be the range of temperature. Hence places like Mumbai,

Chennai, Kolkata and others will have an equable climate as they experience the moderating influence of the sea, while places further inland like Pune, Indore, Hyderabad, Amritsar, Delhi and others will have a continental type /extreme type of climate.

[Any relevant examples]

Page 28: CHAPTER INTERPRETATION OF TOPO-GRAPHICAL MAPS

P-28 G E O G R A P H Y - X

(d) Varied Relief- When mountains lie perpendicular to onshore winds, heavy rainfall occurs on the windward side and scanty rainfall occurs on the leeward side; and when mountains lie parallel to rain-bearing winds, little rain falls. E.g Aravallis – Thar / Rajasthan./ So places that lie on the windward side of mountains like Mumbai, Cherrapunji and others will receive more rain than places that lie on the leeward side of mountains like Pune, Shillong and others. Alignment of mountain ranges/ Presence of Himalayas- Checking the South West Monsoons from blowing into

Central Asia / Checking Cold winds from Central Asia. Impact of prevailing winds / Upper air circulation / Jet Stream. [ICSE Marking Scheme, 2015] [Any 2 points]

Ans.2 The direction is South West. The intense heat during summers in India develops low pressure over the Northern Plains and high pressure over the sea. This low pressure attracts the moisture bearing winds from the southern hemisphere. After crossing the equator, they are deflected to the right and blow over India as the south west monsoon winds.

Ans.3 The cyclonic winds originating over the Mediterranean Sea are called Western Disturbances. These winds blow towards India in the month of November and December and bring heavy rainfall over the North West part of India.

Ans.4 (i) Kerala lies to the SW and hence it is the first state to receive rain from SW Monsoon of the Arabian Sea branch as these winds move from SW to NE. These winds withdraw from North and move towards south and hence Kerala is the last state from which these winds withdraw. Thus, Kerala is the first state to receive the South-West Monsoon and the last to see it withdraw.

(ii) The Konkan coast lies on the windward side of the Western Ghats and hence receives heavy rain from the SW Monsoon winds from the Arabian Sea branch. The windward side of an area is the slope of a range that receives heavy rainfall.

(iii) Kanpur lies to the interior, far away from the influence of the sea, whereas Chennai lies along the coast and is influenced by the sea. Thus, Kanpur is deprived of the moderating influence of land and sea breeze. [ICSE Marking Scheme, 2015]

Ans.5 (i) Patna gets a much heavier rain than Delhi does because Patna lies in the lower Ganga valley while Delhi lies in the upper Ganga valley and the Bay of Bengal branch of the South West

Ganga valley. The South west monsoon Bay of Bengal branch deflects when it comes in contact with the Garo Khasi and Jaintia Hills in Assam moving up the Ganga Valley. The rain shed while following the line of the.

(ii) Western Rajasthan gets no rain from the Arabian Sea Branch of the South West Monsoon winds because it blows parallel to the Aravalli mountains and therefore shed no moisture.

(iii) It never gets too cold in Mangalore, not even in December because of the moderating influence of the land and sea breezes and because it is quite close to the equator. [ICSE Marking Scheme, 2014]

ll

WORKSHEET-17Ans.1 (i) India lies in the tropical belt.

(ii) The climate is influenced by the monsoon winds which are largely confined to the Tropics, i.e. between 20°N and 20°S.

Ans.2 Two main features of the Indian monsoon-(i) Monsoon is erratic and sporadic.(ii) Monsoon is unevenly distributed.

Ans.3 Tropical Cyclones Temperate Cyclones

1. These cyclones are thermal in origin. 1. These cyclones are frontal in origin.

2. The wind velocity of these type of cyclones are very high, i.e., 300km/hr.

2. The wind velocity in this type of cyclones are very low, i.e. 35 to 70 km/hr.

3. These cyclones are active in summer season. 3. These cyclones are active in winter season.[Any Two]

Ans.4 Due to location, vast extent of the country, presence of the lofty Himalayan Mountain, large coastline along the eastern and western sides and the Western Disturbances, there is a great variation in the climate of the Indian Sub-Continent.

Page 29: CHAPTER INTERPRETATION OF TOPO-GRAPHICAL MAPS

P-29S O L U T I O N S

Ans.5 (i) Punjab and Uttrakhand.(ii) The Western disturbances or cyclonic depressions originating over the Mediterranean Sea.

Ans.6 (i) South West Monsoon winds are responsible for the rainfall experienced over the greater part of the Indian Sub-Continent.

(ii) From June to September. Ans.7 (i) Shillong lies on the rain shadow area or the leeward side of the Khasi Hills while Cherrapunji lies on

the windward side.(ii) The moisture laden winds coming from the Bay of Bengal branch gets entrapped in the Garo-Khasi

Hills and causes heavy rainfall. Ans.8 (i) During the Cold weather or winter season the North East Trade Winds dominates the Indian Sub-

Continent.(ii) The North East Trade Winds get completely reversed in the Summer Season. The four months covered by this season are October, November, December and January.

Ans.9 (i) It is so because Tamil Nadu lies in the rain shadow region of Arabian Sea branch and the Bay of Bengal is parallel to the coast.

(ii) It is so because the northern Plains are located far away from the influence of the sea.(iii) Central Maharashtra receives little rainfall because it lies in the rain shadow region of the Western

Ghats. Ans.10 (i) Shimla is situated at a higher altitude than Delhi. Thus, due to lapse rate, as one moves up the altitude,

the temperature decreases. (ii) The cold winds from the north are prevented from entering into India due to the presence of the lofty

Himalayas and thus helps to keep the temperature of the northern plains at a moderate level.(iii) Kochi is a coastal city and is influenced by the sea while Agra is situated in the interior part of the

country experiencing continental type of climate. Thus, Kochi has a lesser annual range of temperature than Agra.

Ans.11 (i) The South West monsoon winds of the Bay of Bengal branch while advancing towards North West sheds heavy rainfall on Patna as it falls first on its way and then the amount of rainfall decreases by the time it reaches Varanasi.

(ii) The Aravalli Hills lie parallel to the South West Monsoon winds coming from the Arabian Sea and thus cannot shed any moisture in Western Rajasthan.

(iii) India has varied climatic conditions because of large latitudinal and longitudinal extent, altitude and distance from sea.

Ans.12 (i) The mountain slopes are cooler than the plains during summer due to normal lapse rate i.e. with every 165 metres of ascent there is a fall of 1°C of temperature.

(ii) Mumbai is warmer than Kanpur in December because Mumbai is a coastal city and has the influence of sea which results in moderate climate.

(iii) Rajasthan receives very little rainfall because Aravalli Hills lie parallel to the South West Monsoon winds coming from the Arabian Sea branch.

ll

Page 30: CHAPTER INTERPRETATION OF TOPO-GRAPHICAL MAPS
Page 31: CHAPTER INTERPRETATION OF TOPO-GRAPHICAL MAPS

P-31S O L U T I O N S

DRAMA

1CHAPTER

4 WORKSHEET-18Ans.1 (i) Black soil is formed by the weathering of igneous rock.

(ii) Cotton. Ans.2 (i) Alluvial soil.

(ii) Alluvial soil is formed by the deposition of sediments and silts brought down and deposited by the rivers.

Ansa Ans.3

Alluvial Soil Black Cotton soil

1. These soils are formed by the deposition of silts and sediments brought down by the rivers.

1. These soils are formed by the weathering of lava flow rocks or igneous rocks.

2. The soil is very fertile and rich in mineral nutrients like potash and lime.

2. The soil becomes sticky and needs to be tilled immediately after the first rain..

Ans.4 Both are red in colour because of the presence of iron oxide / both are infertile soils / both are friable / both are not moisture retentive /both are poor in organic matter. [Any two]

[ICSE Marking Scheme, 2016]

Ans.5 (i) Black soil- moisture-retentive/self-ploughing/rich in lime, iron, calcium, alumina, potash/ deep and fine grained / clayey … [Any one point]

(ii) Red soil- rich in iron/porous/friable/does not get water-logged/responds to manures or fertilizers /rich in potash … [Any one point]

[ICSE Marking Scheme, 2015]

Ans.6

Alluvial Red

Silt and sediments brought down by rivers - transported soil

Formed by weathering of metamorphic rocks - in situ soil

Colour ranges from yellow to brown Red because of iron oxides

Loamy Sandy and friable

Fertile Infertile, responds to fertilizers

Rich in potash and lime Contains soluble salts

Found in northern plains and coasts Along the river banks

Found in South India in the interior Parts of Bihar, Jharkhand, NE states/Orissa/West Bengal.

Ans.7 Formed in-situ as a result of leaching under typical monsoonal conditions with high temperature and heavy rainfall with alternating wet and dry spells.

Ans.8

Bhangar Khadar

1. Older Alluvium soil. 1. Newer Alluvium soil.

2. It is found above the flood levels of rivers and is of a terrace like structure.

2. It lies below the flood levels.

3. It is composed of lime nodules or kankar and has a clayey composition.

3. It is light in colour and is composed of newer deposits.

4. It is not very fertile. 4. It is fertile and is formed of fine silt and clay.

SOIL RESOURCES

Page 32: CHAPTER INTERPRETATION OF TOPO-GRAPHICAL MAPS

P-32 G E O G R A P H Y - X

Ans.9

Alluvial soil of the Northern Plains Alluvial soil on the Coastal Plains

1. This soil has been deposited by the three important river systems-Indus, Ganga and Brahmaputra.

1. This soil is formed by the rivers Godavari, Krishna, Narmada and Tapti.

2. These soils are porous, coarse in the upper region, contains minerals especially potash and light in colour.

2. These soils are non-porous, fine, contains minerals like iron and calcium and are black in colour.

Ans.10 (i) Maharashtra (ii) Gujarat Regur soil help agriculture due to the following qualities :(i) Regur soil has the quality of self-ploughing. (ii) Due to the occurrence of deep cracks during period it helps in oxygenation. (iii) It is moisture retentive. [Any one point]

Ans.11 (i) Laterite soil.(ii) Laterite soils are formed in-situ as a result of leaching under typical monsoonal conditions with high

temperature and heavy rainfall with alternating wet and dry spells.(iii) This soil is not suitable for cultivation because it is highly acidic in nature.

Ans.12 (i) Alluvial soil(ii) Laterite soil(iii) Alluvial soil

Ans.13 (i) Alluvial soil is coarse in the upper valley of the rivers because the eroded matter is carried away by the fast flowing river but in the lower course, the river reduces its speed and the soil particles become finer due to attrition or because the load itself gets eroded.

(ii) Black soil does not get leached because it is clayey and sticky and moisture retentive and therefore the rain cannot wash out the silicates.

(iii) Khadar is the newer alluvium which keeps getting replenished by the river bringing down more eroded material. [ICSE Marking Scheme, 2016]

Ans.14 (i) Pedogenesis- the process of soil formation.(ii) Humus- the decayed organic matter that helps make soil fertile.(iii) Bhangar- the older,less fertile alluvial soil. [ICSE Marking Scheme, 2015]

Ans.15 (i) Laterite soil(ii) Alluvial soil(iii) Laterite soil

Ans.16

Transported Soil In Situ Soil

1. These soils are formed by the deposition of silts and sediments brought down by the rivers, e.g. Ganga, Yamuna, Brahmaputra, etc.

1. It is formed in their original position by the breaking up of parent rocks, e.g. Black Soil, Red soil, Laterite Soil, Desert Soil etc.

2. The soil is very fertile and rich in mineral nutrients like potash and lime.

2. It is less fertile and is deficient in phosphorous, lime, nitrogen and humus.

3. It is found in river valleys. 3. It is found in the Deccan Trap.

Ans.17 (i) Karnataka and Tamil Nadu (ii) 1. Wheat and rice are well grown on this soil. 2. Responds to manures or fertilizers. 3. Rich in iron and potash. [Any two points](iii) Wheat and Rice.

Ans.18 (i) It is fine textured and clayey in nature. (ii) Its moisture retentive and becomes sticky when wet.(iii) Rich in lime, magnesium and iron.

Page 33: CHAPTER INTERPRETATION OF TOPO-GRAPHICAL MAPS

P-33S O L U T I O N S

Ans.19 (i) Laterite soil is not suitable for cultivation because it is highly acidic in nature and is not moisture retentive.

(ii) Red soils are red in colour due to high content of iron oxide present in it.(iii) Khadar soils are preferred to Bhangar soils because Khadar soils are fertile due to fine silts and clay as

they are replenished every year by floods. Ans.20

Alluvial soils in the Lower Courses Alluvial soils in the Upper Courses

1. Soil is fine. 1. Soil is coarse.

2. It is moist. 2. It is dry.

3. It is rich in humus. 3. It is sandy and porous.

ll

WORKSHEET-19Ans.1 (i) Laterite soil is highly acidic in nature.

(ii) It is a leached soil and thus, infertile.(iii) Rich in iron and poor in silica, lime, nitrogen and humus.(iv) It is porous and coarse in texture. [Any two points]

Ans.2 (i) Laterite soils.(ii) Alluvial soils.(iii) Black soil, Regur soil or Black cotton soil.(iv) Black soil.

Ans.3 Laterite soil.(i) Kerala(ii) Odisha

Ans.4 (i) It is porous, friable and coarse.(ii) It does not retain moisture.(iii) Rich in iron and potash.(iv) Deficient in lime, nitrogen, phosphorous and humus.(v) Responds to manures or fertilizers.(vi) It is red in colour due to presence of iron oxides. [Any two points]

Ans.5 Laterite soils are formed in-situ as a result of leaching under typical monsoonal conditions with high temperature and heavy rainfall with alternating wet and dry spells.This soil is not suitable for cultivation because it is highly acidic in nature and is not moisture retentive.

Ans.6 (i) Red soil is red in colour due to high iron oxide content.(ii) Large tracts in Maharashtra are covered with Black soil because it is formed from weathered lava rocks

which have solidified during volcanic activity. Ans.7 Maharashtra. Two crops grown in this soil are Cotton and Sugarcane. Ans.8 With the help of growing leguminous crops after harvesting of cereal crops and from the fertilizers,

soil can get nitrogen. Ans.9 This soil is not suitable for cultivation because it is highly acidic in nature and is not moisture retentive.

This soil is found in the Summits of Eastern Ghats and Western Ghats, West Bengal, Odisha. [Any one place]

Ans.10 It is a process by which soluble substances from the soil are removed through percolation.A region, south of the Tropic of Cancer, formed by ‘Leaching’ is the Highland areas of peninsular Plateau.

Ans.11 (i) Black soil is largely found in the Deccan Trap region because it is formed by the denudation of volcanic rocks, found in that region.

(ii) Khadar is composed of newer alluvium deposits and is fertile as it is formed of fine silt and clay.(iii) Soil erosion by wind is common in arid regions because these regions are devoid of vegetation and

moisture due to scanty rainfall and as such the soil is blown away by strong winds. Ans.12 (i) Transported soil : These soils are formed by the deposition of silts and sediments brought down by

the rivers, e.g. Ganga, Yamuna, Brahmaputra, etc.

Page 34: CHAPTER INTERPRETATION OF TOPO-GRAPHICAL MAPS

P-34 G E O G R A P H Y - X

(ii) In situ : It is formed in their original position by the breaking up of parent rocks, e.g. Black Soil, Red soil, Laterite Soil, Desert Soil etc.

(iii) Humus : It is the organic matter present in the soil formed by the decomposition of plants and animals that helps to make soil fertile.

Ans.13 Two characteristics of Black soil :(i) It is fine textured and clayey in nature.(ii) It has a self-ploughing quality.(iii) Its moisture retentive and becomes sticky when wet and forms crack when dry.(iv) Rich in lime, magnesium, potash, aluminium and iron.(v) Poor in phosphorous, nitrogen and organic matter. [Any two points] This soil is agriculturally important because it is moisture retentive and is rich in minerals like lime,

magnesium, potash, iron and aluminium.Ans.14 These soils are formed by the deposition of silts and sediments brought down by the rivers.

This soil is agriculturally important because it is rich in potash, lime and humus of the Ganga deltaic region.Ans.15 Red soil is formed by the weathering of old hard crystalline and metamorphic rocks. The reasons for

its low productivity is due to deficiency of lime, nitrogen, phosphorous and humus in the soil.Ans.16 Regur soil is formed by the weathering of lava flow rocks or igneous rocks.

Four important properties of Regur soil are : (i) It is fine textured and clayey in nature. (ii) It’s colour vary from deep black to chestnut brown.(iii) Its moisture retentive and becomes sticky when wet and forms crack when dry.(iv) Rich in lime, magnesium, aluminium and iron.

ll

WORKSHEET-20Ans.1 (i) Sheet erosion- The foothills of Shiwalik range.

(ii) Gully erosion- Chambal Valley region.

Ans.2 Removal or destruction of top soil is called soil erosion. Terrace farming / contour ploughing / crop rotation etc. [ICSE Marking Scheme, 2016]

Ans.3 (i) Leaching(ii) Gully Erosion [ICSE Marking Scheme, 2015]

Ans.4 Any two of the following: Terrace farming / contour ploughing Strip cropping Preventing over grazing Planting shelter belts and trees Constructing dams and plugging gullies Introducing better methods of cultivation, use of fertilizers, rotation of crops [ICSE Marking Scheme, 2014] Ans.5 (i) Contour Ploughing : Contours act like bunds. Ploughing along contours on a slope prevents soil being

washed away by rainwater or by surface run-off.(ii) Constructing Dams : Rivers cause soil erosion, therefore, dams are built in the upper course to control

the erosion. Ans.6 Soil Conservation is the effort made by man to prevent soil loss from erosion or reduced fertility

caused by over usage.Re-afforestation is planting of trees in lieu of the number of trees being cut. It helps as the roots of the trees planted again hold the soil tightly and prevents it from eroding.

Ans.7 (i)

Sheet Erosion Wind Erosion

It occurs on gentle slopes where the thin layer of top soil is removed due to rain water.

It takes place where there is less or no vegetation due to high velocity an strong movement of winds and deposition of soil particles.

(ii) Man is largely responsible for soil erosion because he is involved in deforestation, overgrazing of domestic animals, shifting agriculture and faulty farming practices.

Page 35: CHAPTER INTERPRETATION OF TOPO-GRAPHICAL MAPS

P-35S O L U T I O N S

Ans.8 (i) The removal or destruction of the top soil is called soil erosion.(ii) Soil erosion is caused due to increasing population pressure on land, overgrazing by domestic animals,

floods, deforestation and bad farming techniques.(iii) (a) Scheme of Integrated Watershed Management. (b) Scheme for Reclamation and development of ravine areas. (c) Scheme for Control of Shifting Cultivation. (d) National Project on Development and use of Bio-Fertilizers and National Project on Quality

Control implemented. (e) Rainwater Harvesting.

Ans.9 (i) Sheet erosion is the slow removal of a thin layer of soil by rainwater washing it away.(ii) Soil conservation refers to the efforts made to prevent soil from getting eroded.(iii) In situ means to develop in one area without any movement. It refers to residual soil.

[ICSE Marking Scheme, 2016]

Ans.10 (i) As it contains minerals like iron, potash, lime../is loamy/has sufficient depth/is renewed annually/ It is a transported soil which brings along lots of minerals … [Any one point]

(ii) As soil supports all plant life/to increase our agricultural output/ Various methods/ Efforts made by man to check soil erosion and retain the fertility of the soil… [Any one point]

(iii) As it helps prevent soil erosion/holds the soil together/helps maintain the ecological balance/checks global warming/reduces severity of drought… [Any one point]

[ICSE Marking Scheme, 2015]

Ans.11 (i) Different regions in India have different kinds of soil because of the different rocks which get weathered to form soil and the different agents of erosion minerals present / difference in temperature and rainfall.

(ii) Black soil is suitable for the growth of cotton because it is moisture retentive, has self- ploughing qualities and is rich in lime, iron, potash, calcium, claying in nature.

(iii) Soil is a natural resource which must be conserved or else the land becomes barren and cannot be cultivated, food crops will have to be imported and the agro-based industries will suffer. Some kinds of soil are also required for construction purposes. It takes very long for an inch of top soil to be formed. [ICSE Marking Scheme, 2014]

Ans.12 Two important agents of erosion are- Running Water and Wind.(i) Running water- It can be controlled by methods like contour ploughing, terracing, plugging gullies

and planting trees.(ii) Wind- It can be controlled by the methods like strip cropping to check the impact of the winds, by

planting trees along the edges of the fields, waste lands and on steep slopes. Ans.13 Soil Conservation is the preventing of soil loss from erosion or reduced fertility caused by over usage.

Two farming techniques which help in soil conservation are-(i) Contour Ploughing(ii) Terrace Farming.

Ans.14 India is an agricultural country and its economy depends on it. So soil conservation is needed to check floods, soil erosion, etc. Two methods of soil conservation are-

(i) Contour ploughing(ii) Planting trees or Afforestation.

Ans.15

Sheet Erosion Gully Erosion

1. It is the slow removal of a thin layer of soil when vegetation is destroyed.

1. Due to heavy rainfall deep gullies are formed on the soil and remove the soil nutrients.

2. Rainwater washes away the thin layer of bare soils.

2. It removes nutrients and heavy load of loose soils, making the soil unproductive.

3. It occurs on gentle slopes. 3. It occurs in the Chambal Valley region.ll

Page 36: CHAPTER INTERPRETATION OF TOPO-GRAPHICAL MAPS
Page 37: CHAPTER INTERPRETATION OF TOPO-GRAPHICAL MAPS

P-37S O L U T I O N S

DRAMA

1CHAPTER

5 WORKSHEET-21Ans.1 It is Littoral or Mangrove or Tidal Forests. Sundari tree is found there. Ans.2 Tropical Monsoon Deciduous Forests are the most commercially important vegetation belt in India.

The rainfall range is between 100 cm to 200 cm. Ans.3 Teak.

It has high oil content making it very weather resistant. It has great value as timber and is very heavy, tough and durable. It is broadly used for making furniture, houses and also ships. The bark of the teak tree is considered to be astringent and is used to treat bronchitis.

Ans.4 (i) The trees have long roots which enable them to obtain water from deep underground.(ii) The leaves are small which minimises evaporation and the stems are succulent which helps to conserve

water. Ans.5 (i) Teak

(ii) Sundari

Ans.6 (i) Tropical deciduous forest(ii) Teak / Sal / Shisham / Sandalwood [ICSE Marking Scheme, 2016]

Ans.7 (i) Tropical evergreen forest(ii) Due to heavy rainfall of above 200 cm [ICSE Marking Scheme, 2016]

Ans.8 Any two of the following:(i) Tropical deciduous forests have trees found in pure stands.(ii) Trees shed their leaves in a particular season Provide valuable timber and other forest products.

[ICSE Marking Scheme, 2015] Ans.9 (i) 1. Due to scanty rainfall, the forests have xerophytic vegetation.

2. The trees of the forests are stunted and have large patches of coarse grasses. (ii) An example of desert vegetation-Babool Its uses- It has high medicinal value. It is a source of gum and used as an emulsifier. The twigs and

barks are chewed to prevent Vitamin C deficiency. Its wood is used as fire wood and charcoal and for boat building.

Ans.10 (i) Tropical Evergreen Forests.(ii) Montane Forests.(iii) Tropical Desert Forests.

Ans.11 (i) Mangrove / Littoral forest or Tidal forest(ii) Tropical Evergreen forest / Tropical rain forest(iii) Mountain forest. [ICSE Marking Scheme, 2016]

Ans.12 (i) Deodar.(ii) Sundari.(iii) Mahogany or Rosewood. [ICSE Marking Scheme, 2015]

Ans.13 (i) Sundri - firewood / boat building / construction.(ii) Sandalwood - extraction of sandalwood oil which is used in soaps and cosmetics / handicrafts /(iii) Rosewood – furniture / floor boards / veneer / wagon parts. [ICSE Marking Scheme, 2014]

ll

WORKSHEET-22Ans.1 They are dense and have a variety of trees and shrubs.

Ans.2 Tropical evergreen forests are dense/ Heavy logs make it difficult for accessibility/ Trees are found in mixed stands/ Dense undergrowth… [Any two points]

[ICSE Marking Scheme, 2015]

NATURAL VEGETATION

Page 38: CHAPTER INTERPRETATION OF TOPO-GRAPHICAL MAPS

P-38 G E O G R A P H Y - X

Ans.3 Any two of the following :(i) Provide valuable timber and other forest products.(ii) These forests are not as dense as the tropical evergreen forests. The wood of the trees is not so heavy

as to be difficult to transport.(iii) These trees grow in stands and are therefore easier to exploit for commercial purposes.

[ICSE Marking Scheme, 2014] Ans.4 The characteristic of Tropical Rain Forests are-

(i) They are dense and have a variety of trees and shrubs.(ii) Trees reach at a height of 60 m or above.(iii) Due to thick canopy of trees, herbs and grasses cannot grow.(iv) These forests do not have any fixed period of time for shedding of leave. [Any two points]

Ans.5 (i) They are dense and have a variety of trees and shrubs which makes it difficult to exploit.(ii) Due to lack of proper transportation facilities.

Ans.6 (i) These forests are dense, evergreen and of varying heights.(ii) The trees have long roots submerged under water and have pores which help them to breathe during

high tide. Ans.7 The forests grown along the sea coasts, in wet marshy areas, in river deltas, in tidal or other swampy

areas., e.g., in the areas around the eastern coasts, in the saline swamps of Sunderbans in West Bengal, etc.Sundri and Gorjan are the two typical trees found there.

Ans.8 (i) The delta of the Ganga river - mangrove forests / sundarbans / tidal forests(ii) The windward side of the Western Ghats - tropical evergreen forests(iii) The Deccan Plateau - monsoon deciduous forests/ thorn forests [ICSE Marking Scheme, 2014]

Ans.9 (i) Semul.(ii) Mahogany.(iii) Sandalwood.

Ans.10 (i) 1. Tidal Forests- Sunderbans in West Bengal or Ganga Delta Region. 2. Thorn and Scrub Forests- Rajasthan.

(ii) Thorn and scrub forests are found in this region because these forests receive less than 50 cm of rainfall.

Ans.11 (i) Teak- It has high oil content making it very weather resistant. It has great value as timber and is very heavy, tough and durable. It is broadly used for making furniture, houses and also ships. The bark of the teak tree is considered to be astringent and is used to treat bronchitis.

(ii) Sal- It is hard, tough and heavy wood. It is mainly used for making doors, windows, railings of bridges, beams, railway sleepers.

(iii) Sandalwood- It is widely used in the cosmetic industry. It is used as perfume and its oil is used in making aromatic substances. It is also used for making ornamental objects like statues.

Ans.12 Tropical Evergreen Forests are found in the western part of Western Ghats.Two reasons are-

(i) This region receives an annual rainfall of more than 200 cm.(ii) The trees remain evergreen.

ll

WORKSHEET-23

Ans.1 (i) Forests control humidity, lower the temperature and can be responsible for rain.(ii) The roots of the trees check the speed of running water which then does not overflow its banks and

also because the roots take in some of the water. [ICSE Marking Scheme, 2014] Ans.2 (i) Forest plays an important role in modifying climate and weather.

(ii) They also play an important role in controlling soil erosion by wind and water. ll

Page 39: CHAPTER INTERPRETATION OF TOPO-GRAPHICAL MAPS

P-39S O L U T I O N S

WORKSHEET-24Ans.1

Agro Forestry Social Forestry

1. It is a sustainable land use system that maintains or increases the total yield by combining food crop together with forest tree and livestock ranching on the same unit of land.

1. It is the management and protection of forests and afforestation of barren and deforested lands with the purpose of helping in the environmental, social and rural development.

2. It aims to provide soil conservation to improve the growth of forest products and agricultural crops together.

2. It aims at raising plantations by the common man so as to meet the growing demand for timber, fuel wood, fodder, etc., thereby reducing pressure on traditional forest areas.

Ans.2 The reasons are as follows :

(i) Land was cleared for cultivation, industrialisation and urbanisation.(ii) Shifting agriculture was practised.(iii) Forest was cut recklessly for raw material. [ICSE Marking Scheme, 2016]

Ans.3 Afforestation Deforestation

Planting of trees over a large area Cutting of trees over a large area

Disadvantage : Leads to soil erosion/extinction of flora and fauna/rise in temperatures/floods/increases severity of drought helps maintain the ecological balance/checks global warming/reduces severity of drought… [Any one point]

[ICSE Marking Scheme, 2015]

Ans.4 (i) To maintain the environmental stability.(ii) To preserve natural forests through wide variety of flora and fauna.(iii) To checksoil erosion.

Ans.5 Three methods for the Conservation and Development of Forests :(i) To plant trees or Afforestation.(ii) Banning the practice of shifting cultivation prevalent amongst the tribals.(iii) To discourage cutting of trees and ensure to plant ten saplings in lieu of felling one tree.

ll

Page 40: CHAPTER INTERPRETATION OF TOPO-GRAPHICAL MAPS
Page 41: CHAPTER INTERPRETATION OF TOPO-GRAPHICAL MAPS

P-41S O L U T I O N S

DRAMA

1CHAPTER

6 WORKSHEET-25Ans.1 Tamil Nadu, Karnataka, Rajasthan, Maharashtra, etc. [Any two states]Ans.2 (i) High demand of water due to increasing population is leading to the lowering of the ground water

levels.(ii) Rainfall in India is seasonal, erratic and unreliable and thus the farmers cannot wholly depend on

rain.(iii) More than 90% of water is utilized for irrigation.(iv) The Industries also utilizes a lot of water and pollutes it too. [Any two points]

Ans.3 Two main reasons why water scarcity occurs in India-(i) Increasing demand of water for domestic and industrial uses.(ii) Lack of Watershed Management.(iii) Rainfall is erratic and unreliable. [Any two]

Ans.4 (i) Rainwater Harvesting(ii) 1. Harvesting surface and groundwater, 2. Prevention of losses through evaporation and seepage, 3. Other techniques aimed at conservation and efficient utilization of limited water endowment.

[Any two points]

Ans.5 (i) It is a technique of increasing the recharge of ground water by capturing and storing rain water.(ii) To make water available for future use. [Any two points](iii) To avoid flooding of roads. [ICSE Marking Scheme, 2016]

ll

WORKSHEET-26

Ans.1 Rainwater harvesting helps to meet the increasing demand for water/ helps to check surface run-off that chokes the drain / recharge groundwater

Ans.2 Two advantages of rainwater harvesting :(i) Prevents wastage of water.(ii) Reduces soil erosion.(iii) Recharge groundwater resource. [Any two points]

Ans.3 (i) Severe water shortage is due to wastage / pollution/ overuse/ mismanagement / Increasing population / increasing Industries. [Any two points]

(ii) make rainwater harvesting mandatory in every housing colony/levy fine on wastage/pass laws to ensure water is recycled in factories. [Any two points]

[ICSE Marking Scheme, 2015]

Ans.4 Any three of the following :(i) The growth of population has resulted in water scarcity.(ii) Deforestation has led to drought and reduction of rainfall.(iii) The underground water has been exploited and the water table has been Lowered.(iv) Much of the water is used up for cultivation Water is required also for industries.(v) Much of both the ground water and the surface water has got polluted.

[ICSE Marking Scheme, 2014] Ans.5 (i) Two methods of water harvesting in India-

1. Surface runoff harvesting2. Roof Top-Water harvesting.

WATER RESOURCES

Page 42: CHAPTER INTERPRETATION OF TOPO-GRAPHICAL MAPS

P-42 G E O G R A P H Y - X

(ii) 1. Harvesting surface and groundwater.2. Prevention of losses through evaporation and seepage.3. Techniques aimed at conservation and efficient utilization of limited water endowment.4. To make water available for future use. 5. To avoid flooding of roads and improve ground water quality. [Any two points]

Ans.6 (i) Roof Top Water harvesting(ii) Surface runoff harvesting(iii) Khul and Johads in Western Himalayas and in Rajasthan respectively.

ll

WORKSHEET-27Ans.1 Perennial canals are more useful than inundation canals because perennial canals contain water

throughout the year and never dry up while inundation canals are seasonal. Ans.2 Uttar Pradesh Ans.3 (i) Significance of Irrigation in Punjab-

1. Availability of water is highly uneven and variable. 2. It is an agricultural state and along with wheat rice is cultivated at a large scale. (Any one)

(ii) Significance of Irrigation in Rajasthan- 1. Rajasthan lies in the semi-arid region with less than 25 cm of rainfall.2. The groundwater level is available only at a depth of 30 to 60 m. [Any one]

Ans.4 Rain occurs mainly in four months Some crops like rice and sugarcane need more water. To cater to increasing demand of food and cash crops there is a need to grow crops all the year round.

[ICSE Marking Scheme, 2016]

Ans.5 Fertile agricultural land Cheap electricity Large amount of underground water. [ICSE Marking Scheme, 2016]

Ans.6 Irrigation is required as rainfall is erratic and unevenly distributed, Irrigation is required to grow crops throughout the year as rainfall is seasonal, Irrigation is required as some crops like rice and jute require more water than supplied by rain, Irrigation is required to increase agricultural output by providing the right amount of water at the right time. Multiplicity of crops / Commercialised crops/ Nature of Soil / Monsoon is seasonal. [Any two points]

[ICSE Marking Scheme, 2015]

Ans.7 Modern methods of irrigation- Drip irrigation, Spray irrigation, Sprinkler irrigation. [Any two types] Reason- they prevent wastage of water/provides the right quantity of water at the right time / No

waterlogging/ No soil erosion [Any one reason][ICSE Marking Scheme, 2015]

Ans.8 Well irrigation is still popular as it is cheap/easy to construct/occupies a small area/control of water is in the hands of the farmer / Can be dug at any convenient place/ Independent source of Irrigation.

[Any two points][ICSE Marking Scheme, 2016]

Ans.9 (i) (i) Uncertainty of rainfall.(ii) Uneven distribution of rainfall.(iii) Requirement of different quantities of water for various crops for their growth.(iv) Dependent of crops on the nature of soil.(v) Utilization of river water effectively.(vi) To increase or maximize production. [Any two points]

(ii) Soft soil and flat level land.

Page 43: CHAPTER INTERPRETATION OF TOPO-GRAPHICAL MAPS

P-43S O L U T I O N S

Ans.10 (i) (i) It is inexpensive as they are mostly natural.(ii) It is highly beneficial in the uneven rocky plateau of Deccan since rainfall is seasonal.(iii) It is highly significant because it stores the abundant rainwater and reduces the waste of the

excessive flowing water. [Any one point] Tank irrigation is commonly used in South India like Andhra Pradesh, Telangana, etc.

(ii) Tubewells are deeper wells from where the water is lifted from a great depth of 20-30 m by the use of power pumps.

It is reliable during dry season when the surface water dries up since the tube well is drilled upto the permanent water table.

(iii) It supplies the water to the roots of the plants slowly through pipes, valves, tubing etc. and thus saves water and fertilizer. This helps in the reduction of evaporation.

Ans.11 (i) It is so because it is water saving as there is no loss of water through seepage or evaporation.(ii) It is able to irrigate a larger agricultural land and a large amount of underground water is easily

available.(iii) It is because Northern plains have a good network of perennial rivers which supply water to the

canals throughout the year.

Ans.12 (i) Canal irrigation is popular in Northern plains because it has perennial rivers and land is soft enough for canal to be constructed from the rivers to the fields.

(ii) Being in Deccan region Karnataka has natural depressions and hard sub surface rocks which make tank irrigation important.

(iii) As it helps to conserve water. [ICSE Marking Scheme, 2016] ll

WORKSHEET-28Ans.1 (i) Uttar Pradesh

(ii) Andhra Pradesh. Ans.2 Uttar Pradesh and Punjab. Ans.3 It is less expensive and can be dug anywhere where the soil is soft.

Ans.4 Deccan Plateau is made up of hard impermeable rock which does not allow the rainwater to percolate underground. The Deccan Plateau is highly dissected and so has many natural hollows and depressions. [ICSE Marking Scheme, 2014]

Ans.5 Surface wells rely on ground water and not on rivers to flood. They can be dug anywhere on the farm and the farmer will have water even if he is not too close to a river/cheaper.

[ICSE Marking Scheme, 2014]Ans.6 Two states- Uttar Pradesh and Punjab Its importance-

(i) Less expensive and a reliable source of irrigation.(ii) Can be dug anywhere where the soil is soft. [Any one]

Ans.7 Inundation canals are converted to perennial canals due to the following reasons-(i) Inundation canals are non-perennial and dependent on rain.(ii) It cannot supply water on demand or as and when required.(iii) These type of canals can irrigate only low land areas. [Any two points]

Ans.8 Two disadvantages of tank irrigation are-(i) In the absence of rainwater during dry season, the tanks become dry and fail to provide water for irrigation.(ii) Due to deposition of sediments, the tanks get silted up soon and desilting is necessary for making

irrigation suitable which is expensive.(iii) Tanks occupy large fertile areas which otherwise could be used for agricultural purposes. [Any two]

Ans.9 (i) Large amount of underground water is easily available.(ii) It is reliable during dry season when the surface water dries up since the tube well is drilled upto the

permanent water table.

Page 44: CHAPTER INTERPRETATION OF TOPO-GRAPHICAL MAPS

P-44 G E O G R A P H Y - X

Ans.10 Four disadvantages of tank irrigation in South India-(i) Deccan Plateau is made up of hard impermeable rock which does not allow the rainwater to percolate

underground.(ii) The Deccan Plateau is highly dissected and has many natural hollows and depressions.(iii) In the absence of rainwater during dry season, the tanks become dry and fail to provide water for

irrigation.(iv) Since tanks are very extensive and shallow, huge quantities of stored water go waste as it gets

evaporated or sinks underground.

Ans.11 Any one geographical reason of the following:(i) The rainfall is erratic; It is seasonal It is unevenly distributed over the land Some crops require more water than the amount provided by the rain. Some crops need water during the period when there is no rain.(ii) No loss of water through evaporation. Judicious use of water with no wastage The flow of water can be customised so as to be beneficial to each crop.(iii) Percolation of water through the canals can lead to swamps Alkaline salts from the water table

below can reach the surface and make the soil unproductive Due to water logging the capacity of the soil to absorb water decreases and the water which then

collects can ruin the crop. [ICSE Marking Scheme, 2014] Ans.12 (i) Punjab and Haryana

(ii) Two reasons are :

1. There are a number of perennial rivers.

2. In these states large tracts of land are under cultivation which requires water throughout the year for irrigation.

Ans.13 Two advantages are-

(i) It is able to irrigate a larger agricultural land.

(ii) Large amount of underground water is easily available.

(iii) It is reliable during dry season when the surface water dries up since the tube well is drilled upto the permanent water table. [Any two points]

One disadvantage is-

(i) It is costly as it requires regular supply of electricity.

(ii) Irrigation is not possible if the underground water level is low.

(iii) Excessive use of tubewell leads to lowering of groundwater level. [Any one point]ll

Page 45: CHAPTER INTERPRETATION OF TOPO-GRAPHICAL MAPS

P-45S O L U T I O N S

DRAMA

1CHAPTER

7 WORKSHEET-29Ans.1 Manganese is used to make steel tough, hard and rust resistant and that’s why it is an important raw

material for iron and steel industry.

Ans.2 Keonjhar, Mayurbhanj, Sambhalpur, Sundergarh, Cuttack and Koraput. [Any one] Ans.3 (i) Bauxite is mainly used in aircrafts, automobiles, shipping industry, household appliances, rail wagons,

coaches, etc.(ii) Bauxite is found in Odisha, Goa, Gujarat, Maharashtra, etc. [Any one]

Ans.4 (i) Haematite(ii) Magnetite(iii) Limonite(iv) Siderite.

Ans.5 (i) Iron ore – are used in Steel making / Slag for cement / sludge for fertilizer. [Any one use](ii) Bauxite- to extract aluminium/ light / used in aircraft/ use in automobiles

[ICSE Marking Scheme, 2015]

Ans.6 (i) Aluminium(ii) Used for manufacture of aircraft / utensils / wires. [ICSE Marking Scheme, 2016]

Ans.7 (i) Cement - limestone(ii) Aluminium - bauxite(iii) Synthetics - oil or coal. [ICSE Marking Scheme, 2014]

ll

WORKSHEET-30

Ans.1 Magnetite [ICSE Marking Scheme, 2016] Ans.2 Manganese is used in chemical industries for manufacturing bleaching powder.

Ans.3 Limestone is needed in the manufacture of iron and steel, cement, fertilizers / Fluxing material in smelting of iron ore / glass manufacturing / manufacture of Chemicals – soda ash, caustic soda, bleaching powder / Use in paper / sugar and aluminium. [Any two] [ICSE Marking Scheme, 2015]

Ans.4 Any two of the following: Raw material in the iron and steel industry / to make steel tough and rust proof In the manufacture of black enamel In the chemical industry In the electrical industry In the glass industry In the battery industry To form alloys. [ICSE Marking Scheme, 2014] Ans.5 Bauxite Two uses of aluminium-

(i) Used in automobiles, aircraft(ii) Packaging like cans, foil, etc.

Ans.6 (i) Bauxite (ii) The largest deposits of Manganese are found in Balaghat in Madhya Pradesh.

Ans.7 Odisha is the leading producer of manganese in India.(i) It is used in manufacture of chemical and electrical equipments.

MINERAL RESOURCES

Page 46: CHAPTER INTERPRETATION OF TOPO-GRAPHICAL MAPS

P-46 G E O G R A P H Y - X

(ii) It is used in chemical industries for manufacturing bleaching powder.(iii) It is used in dry cell batteries. [Any two uses]

Ans.8 Haematite, Magnetite, Limonite and Siderite are the different types of iron ore in India. Magnetite is the best quality of iron ore.

ll

WORKSHEET-31

Ans.1 Kemmangundi [ICSE Marking Scheme, 2016] Ans.2 Haematite and Magnetite are mostly mined in India.

Two leading iron producing states are- Karnataka, Odisha, Chhattisgarh, Goa and Jharkhand [Any two]

Ans.3 (i) Odisha : Keonjhar(ii) Jharkhand : Singhbhum

Ans.4 Two states in India where manganese is found are : Odisha, Karnataka, Madhya Pradesh, Maharashtra, Andhra Pradesh, Goa, Telangana, Jharkhand and

Rajasthan. [Any two] One use of manganese :(i) It is used in manufacture of chemical and electrical equipments.(ii) It is used to manufacture coloured glass.(iii) It is used in chemical industries for manufacturing bleaching powder.(iv) It is used in dry cell batteries.(v) Manganese is also used to manufacture vital enzymes for the metabolism of fats and proteins.

[Any one use] Ans.5 Chhattisgarh, Jharkhand and Odisha. [Any two] Ans.6 (i) Two reasons why minerals are important :

1. Minerals are the source of raw materials and form the basis for industries.2. They are the source of energy/power e.g. coal and petroleum.

(ii) Odisha : Keonjhar, Myurbhanj and Sambalpur Chhattisgarh : Bailadillain Dantewada district and Durg district.

Ans.7 India is the largest producer of bauxite in South Asia. Bauxite is considered an important mineral because (i) It is rust resistant, strong and lightweight metal(ii) A good conductor of electricity.(iii) It is mainly used in aircrafts, automobiles, shipping industry, household appliances, rail wagons,

coaches, etc. ll

Page 47: CHAPTER INTERPRETATION OF TOPO-GRAPHICAL MAPS

P-47S O L U T I O N S

DRAMA

1CHAPTER

6 WORKSHEET-32

Ans.1 Raniganj coalfield. Ans.2 Coal is widely used in the iron and steel industries.

Ans.3 Mumbai High. [ICSE Marking Scheme, 2016]

Ans.4 Bituminous coal. [ICSE Marking Scheme, 2016] Ans.5 (i) Mumbai High.

(ii) Barauni refinery.

Ans.6 Anthracite, Bituminous and Lignite. [ICSE Marking Scheme, 2016]

Ans.7 (i) Raniganj in West Bengal(ii) Digboi in Assam [ICSE Marking Scheme, 2016]

Ans.8 (i) Mathura / Guwahati / Digboi / Haldia / Koyali / Barauni / Kochin Chennai / Panipat / Mumbai / Vishalapatnam

(ii) Jharkhand [ICSE Marking Scheme, 2015]

Ans.9 Bituminous is used for domestic purposes. Burns slowly, no smoke, no ash, high heating value. [ICSE Marking Scheme, 2016]

Ans.10 Iron and Steel Industry and the Sugar industry. Ans.11 Coastal oil refinery- Mumbai and Vishakhapatnam Inland oil refinery- Mathura and Barauni. Ans.12 Bituminous coal is popular for domestic use. It contains 80% carbon in it and its calorific value is very

high. Ans.13 Two off-shore oilfields of India are-

(i) Mumbai High.(ii) Cambay Basin.

Ans.14 (i) 1. Petroleum is widely used as fuel for transportation on land, on sea and in the air. 2. It is also used for power generation.

(ii) One new oilfield in India- Aishwarya oilfield in Barmer district in Rajasthan. One old oilfield in India- Digboi.

Ans.15 (i) As port cities are large consumers of oil and its products and by-products/to cut down transport costs as most of the crude oil is imported. [Any one point]

(ii) As petroleum is derived from the decomposition of organic matter under tremendous heat and pressure.

(iii) As it has numerous uses-it can be converted into oil, gas and electricity/ used to generate power/ used as fuel/used as a raw material in numerous industries like iron and steel, fertilizers and cement/ yields valuable by-products like naphtha and phenol. [Any one point]

[ICSE Marking Scheme, 2015]

Ans.16 (i) Coal - Jharkhand(ii) Oil - Maharashtra(iii) Manganese – Odisha [ICSE Marking Scheme, 2014]

Ans.17 (i) Anthracite has a heating value, high caloric value and is smokeless and is thus good for domestic purpose.

(ii) Oil refineries are located close to oil fields or near ports to avoid transportation of mineral oil to the interior places of the country as it is highly inflammable and risky.

(iii) It is so because to manufacture iron and steel, it is used as the important source of energy and is also used as a raw material in the chemical and other industries.

ll

ENERGY RESOURCES

Page 48: CHAPTER INTERPRETATION OF TOPO-GRAPHICAL MAPS

P-48 G E O G R A P H Y - X

WORKSHEET-33Ans.1 Jharia in Jharkhand and Raniganj in West Bengal. Ans.2 Maharashtra is the largest producer of mineral oil. Ans.3 Mathura Refinery and Mumbai Refinery. Ans.4 Coal is considered the most important industrial power resource because coal is found in abundance

and 67% of energy needed in our country is met by coal. Ans.5 Two advantages of natural gas-

(i) It can be stored safely and can be transported efficiently through pipelines, cylinders, etc.(ii) It is reliable and is conveniently used for cooking and for running many appliances.(iii) It is cheaper and cleaner than petrol or diesel. [Any two points]

Ans.6 West Bengal and Jharkhand are the two states with large deposits of coal. Raniganj in West Bengal and Jharia in Jharkhand. Ans.7 Lignite is a soft brown combustible sedimentary rock and has a carbon content of only 40%. Lignite is mined in Neyveli in Tamil Nadu. Ans.8 Jharkhand is an important coal producing state in India. Jharia coal-mine is located in that state. Ans.9 Two important oil fields in India are-

(i) Digboi in Assam.(ii) Cambay Basin in Gujarat.

Ans.10 Two main drawbacks of coal found in India-(i) Greenhouse Gas Emissions is considered as one of the biggest drawbacks of coal energy.(ii) It also results in harmful emission of substances like Mercury and Sulfur Dioxide which cause health

hazards amongst the population of that area.(iii) Uneven distribution of coal. [Any two points]

Ans.11 Gondwana Coal fields and Tertiary coal fields are the two coal fields in India. Gondwana Coalfields- West Bengal Tertiary Coalfields- Assam Ans.12 Two products of an Oil refinery are Gasoline, LPG, naphtha, diesel, kerosene, fuel, lubricating oils,

bitumen, petroleum and coke. [Any two products] Oil refinery along the coast- Trombay and Vishakhapatnam. Oil refinery away from the coast- Mathura and Barauni. Ans.13 Regions which has natural gas deposits are Mumbai High, Assam, Rajasthan, Tamil Nadu and Tripura.

[Any one region] Two uses of Natural Gas-

(i) It is mainly used as a fuel for generating electricity and heat.(ii) Natural gas in compressed form is used as fuel for vehicles which is known as CNG (Compressed

Natural Gas). Ans.14 Oil refineries are located close to an oil field or in a coastal city because-

1. To minimise the cost of transport 2. To avoid transportation of mineral oil to the interior places of the country as it is highly inflammable.

One oil refinery in the private sector is The Reliance Petroleum Limited at Jamnagar in Gujarat. Ans.15 (i) Oldest coal field- Raniganj in West Bengal

Largest coal field- Jharia in Jharkhand(ii) Two raw materials derived from coal-

1. Coke2. Tar

Ans.16 (i) Two uses of mineral oil-1. It is used as a fuel and as a lubricant.2. It is also used in the chemical, pharmaceutical, textile and many other industries.

Page 49: CHAPTER INTERPRETATION OF TOPO-GRAPHICAL MAPS

P-49S O L U T I O N S

(ii) Old mineral producing area- Assam New mineral producing area- Mumbai High

Ans.17 Anthracite, Bituminous, Lignite and Peat are the four types of coal. Bituminous coal is the best for industrial purposes. It is an important ingredient in iron and steel

smelting in blast furnaces. Ans.18 Three disadvantages of conventional sources of energy are-

(i) These are non-renewable sources of energy.(ii) They are non-environment friendly.(iii) These are exhaustible sources.

Ans.19 (i) An impervious rock, that prevents material from leaking out.(ii) Organic material composed of dead animals and plants in sufficient supply.(iii) Right pressure and temperature to allow the organic material to be transformed into hydrocarbons.(iv) Millions of years of time.

Two advantages of using natural gas-(i) Natural Gas is considered to be environment friendly.(ii) It can be stored safely and can be transported efficiently through pipelines, cylinders, etc.(iii) It is reliable and is conveniently used for cooking and for running many appliances.

[Any two points] ll

WORKSHEET-34Ans.1 Multi-purpose projects are vital because these projects are conducive to the economic development of

the country, are boosting agriculture through irrigation, prevent flooding and generate electricity. Ans.2 Two advantages of Hydro-Electric power over Coal and Petroleum-

(i) Hydro-Electric power produces electricity from water and thus non-pollutant.(ii) It is inexhaustible and renewable source of energy.

Ans.3 Bhakra-Nangal Project has been built on the river Sutlej. Punjab, Haryana and Rajasthan are benefitted by this project. Ans.4

Dam Barrage

1. It is built for storage of water in a reservoir to raise the level of water.

1. It is built to divert the river water.

2. It stores surplus flood water and distributes additional water through power and irrigation.

2. There is no storage of water; canals take water directly from the rivers..

Ans.5 Two hazards of constructing dam :

(i) Ecological impact due to the disturbance in the ecology of water.(ii) Human displacement from the place where they have been living for long period of time.

Ans.6 Three advantages that hydroelectric power has over thermal power are : (i) Hydropower is generated from the force of running water while thermal power is produced by

burning coal, petroleum or natural gas.(ii) Hydel power is a clean source of energy and do not cause pollution while thermal produces air and

dust pollution. (iii) Hydel power is inexhaustible and renewable sources of energy while thermal power is non-renewable

sources of energy. Ans.7

Thermal Power Project Hydel Power Project

1. It is produced by burning coal, petroleum, natural gas, etc.

1. It is generated by storing water.

Page 50: CHAPTER INTERPRETATION OF TOPO-GRAPHICAL MAPS

P-50 G E O G R A P H Y - X

2. The initial cost of generating plants is cheaper but maintenance costs are high and need more labour.

2. The initial cost of generating plants is costly but maintenance costs are low and less labour needed.

3. It causes pollution. 3. It is clean and non-polluting.

4. These are non-renewable and exhaustible sources of energy.

4. These are renewable and inexhaustible sources of energy.

ll

WORKSHEET-35Ans.1 Methane gas is generated in bio-gas plants. Biogas is a non-polluting, cheap and clean source of energy. Ans.2 Solar Energy is the most widespread non-conventional source of power. India is a tropical country and receive ample amount of solar energy throughout the year. It is also

non-polluting and inexpensive. Ans.3 Bio-gas is increasingly being used because-

(i) Biogas is a cheap and non-polluting.(ii) It generates employment in the rural areas.(iii) It is renewable source of energy.(iv) It significantly lowers the greenhouse effects on the earth’s atmosphere. [Any two points]

Ans.4 Nuclear power generation carried out on a small scale in India because :(i) It is expensive.(ii) There is shortage of raw materials like uranium and thorium.(iii) There is a problem of radioactive waste disposal.

ll

Page 51: CHAPTER INTERPRETATION OF TOPO-GRAPHICAL MAPS

P-51S O L U T I O N S

DRAMA

1CHAPTER

9 WORKSHEET-36Ans.1 (i) Inadequate use of manures and fertilizers, negligence of crop rotation, use of poor quality seeds,

inadequate water supply, etc. leads to low productivity.(ii) Use of simple and old agricultural tools, use of no or less machines for ploughing, sowing, irrigating,

pruning, harvesting and threshing results in low yield. Ans.2 The consolidation of landholdings helped the farmers in the following ways :

(i) It saved time, energy and money in moving from one farm to another.(ii) Farmers feel encouraged to spend money on the improvement of his land. [Any one point]

Ans.3 – Provides food for our growing population(i) Provides fodder for livestock(ii) Provides employment(iii) Supplies raw material for agro-based industries(iv) Gives rise to industries related to agriculture, like pesticides, fertilizers, farming tools(v) Earns foreign exchange through export. [ICSE Marking Scheme, 2016]

[Any two]

Ans. 4 Farm fragmentation reduces the size of the farm on which it is not possible to use harvesters or other farm machinery and modern methods of agriculture. Because of poverty the farmers cannot afford good quality seeds, fertilizers or pesticides or farm implements and so the yield is low.

[ICSE Marking Scheme, 2014]

Ans.5 Problems of Agriculture in India-(i) Lack of adequate irrigation facilities and dependence on monsoon.(ii) The land holdings are uneconomic due to their small size and as such the yields are low.(iii) Agriculture is becoming mechanized and requires huge capital investments to purchase machineries,

fertilizers, pesticides and high yielding variety seeds. The Indian farmers are poor to buy all these materials.

(iv) A majority of Indian farmers are still dependent on the primitive and poor techniques of producing crops. [Any two points]

ll

WORKSHEET-37Ans.1 (i) Plantation farming is an extensive system of agriculture in which single cash crop is cultivated on a

large scale in an estate.(ii) Crops like tea, coffee, rubber, spices, etc. are grown under plantation farming mainly for profit.

Ans.2

SUBSISTENCE FARMING COMMERCIAL FARMING

1. It is a self-sufficient farming in which the farmers grow enough food to feed himself and his family.

1. It is a farming where crops are grown and animals are reared for sale in the market for commercial purposes.

2. The farmer uses simple and primitive tools with traditional method of agriculture.

2. It largely depends on machines, uses HYV seeds, chemical fertilizers, pesticides and insecticides to obtain higher yield.

Ans.3 Mixed Farming is a combination of growing crops and rearing of cattle simultaneously. The main benefit of this type of farming is that it ensures a steady income for the farmers because if any one business or farming fails, the other means can support.

AGRICULTURE I AND II

Page 52: CHAPTER INTERPRETATION OF TOPO-GRAPHICAL MAPS

P-52 G E O G R A P H Y - X

Ans4. (i) Shifting cultivation involves a patch of forest which is cleared, trees are cut down and the stumps set on fire. The patch is then cultivated for a few years and when the soil becomes infertile, the cultivator moves to a fresh piece of land and repeats the same procedure.

(ii) The bud from a good rubber tree is grafted on the seedling of a new rubber tree, Once the bud starts sprouting the shoot from the seedling is cut down and the bud then grows into a tree with all of the traits of the mother tree. [ICSE Marking Scheme, 2016]

Ans.5 Any two of the following: Huge capital investment Large land holding Labour intensive Single crop or monoculture Crop grown usually for export Crops grown as plantation crops are tea, coffee, rubber and spices. [ICSE Marking Scheme, 2016]

Ans.6

INTENSIVE FARMING EXTENSIVE FARMING

1. Intensive farming is a system of farming that involves higher input of labour, increased use of fertilizers, pesticides, high quality seeds, etc.

1. Extensive farming uses machinery and scientific methods to produce large quantity of crops.

2. It is labour intensive farming. 2. It is highly capital intensive.

Ans.7 Characteristics of plantation farming-(i) Plantation farming is an extensive system of agriculture in which single cash crop is cultivated on a

large scale in an estate.(ii) Crops like tea, coffee, rubber, spices, etc. are grown under plantation farming mainly for profit.(iii) This type of farming is practiced in vast lands extending from a few hectares to thousands of hectares.(iv) Modern methods, techniques and machineries are used for growing crops.(v) Huge capital is invested in buying machineries, fertilizers, pesticides and building factories for

processing of crops. [Any two points] One important plantation crop-Tea.

Ans.8

SHIFTING AGRICULTURE PLANTATION AGRICULTURE

1. Shifting agriculture is also known as “slash and burn method”. It is a primitive method of farming in which a patch of forest is cleared by felling trees or by burning the trees.

1. Plantation farming is an extensive system of agriculture in which single cash crop is cultivated on a large scale in an estate.

2. The crops grown in this type of farming are- maize, millets, barley, buckwheat, root crops, rain fed rice and vegetables.

2. Crops like tea, coffee, rubber, spices, etc. are grown under plantation farming mainly for profit.

Ans.9 Two main features of Subsistence Agriculture :(i) Subsistence farming is a self-sufficient farming in which the farmers grow enough food to feed himself

and his family.(ii) The farmers have small land and do not use fertilizers and thus the yield is low.(iii) The output is mostly for local requirements with little or no surplus trade.(iv) The land holdings are small and scattered.(v) The farmer uses simple and primitive tools with traditional method of agriculture.

[Any two points] ll

WORKSHEET-38Ans.1 Sowing and harvesting are the two methods of growing wheat.

Page 53: CHAPTER INTERPRETATION OF TOPO-GRAPHICAL MAPS

P-53S O L U T I O N S

Ans.2 Millets are known as dry crops because they can grow in areas of low rainfall ranging from 50-100 cms.

Ans.3 Pulses are high in protein and are an important vegetarian diet. [ICSE Marking Scheme, 2015] Ans.4 In this method after ploughing, the seeds are scattered all over the field by hand before the onset of

monsoon. Ans.5 (i) Transplantation is the process in which seedlings are first grown in nurseries and after 4 to 5 weeks

when the saplings attain a height of 25 to 30 cm they are transplanted to prepared rice fields.(ii) Transplantation enables to select only healthy seedlings for the plants.

Ans.6 Drilling- In this method the seeds are sown in the furrows with the help of a drill made of a bamboo.Ans.7 Pulses are grown as rotation crops as they are leguminous crops that fix atmospheric nitrogen in the

soil and increase the natural fertility of the soil. Ans.8 Kharif Season and Rabi Season are the two main crop seasons in India.

Kharif Crops : Rice, jowar, sugarcane, bajra, ragi, maize, cotton and jute. Rabi Crops : Wheat, barley, rapeseed, linseed, gram, peas, mustard.

Ans.9 (i) Rice needs flooded fields and the subsoil of clay does not allow the water to percolate.(ii) By growing rice in nurseries, we save water, and the plants can be transplanted in rows to facilitate

the use of pesticides and harvesting. [ICSE Marking Scheme, 2016]

Ans.10 Rabi Crop Kharif CropSown in Oct-Nov and harvested in Mar-April / Sown in June and harvested in Oct-Nov/Winter crop Summer crop

[ICSE Marking Scheme, 2015]

Ans.11 (i) West Bengal, Punjab, Uttar Pradesh and Andhra Pradesh. [Any two states] (ii) Advantages of growing rice in nurseries :

1. It enables deeper penetration of the roots in the soil.2. Less wastage of seeds.3. It minimizes weed pressure by resowing.4. It gives higher yield. [Any two points]

Ans.12 Temperature required- 10°C-15°C. Rainfall- 50 cm to 100 cm Soil- well-drained loams and clay loams.

Ans.13 (i) Rice. It enables deeper penetration of the roots in the soil and gives higher yield.(ii) 1. Temperature- 18°C - 32°C 2. Rainfall- 150 cm - 300 cm 3. Soil- Deep fertile clayey or loamy soils.

Ans.14 (i) (1) Kharif crops are sown in the months of June and July and (2) harvested in September and October.(ii) Cotton.

Ans.15 Wheat is grown in the Rabi season in northern India. Rainfall- 50 cm to 100 cm Soil- Alluvial soil or well-drained loamy and clay loams soil.

Ans.16 Wheat is a Rabi crop because :1. Sown in October- November and harvested in January in south, by March-April in north.2. Temperature of 10°C-15°C is suitable for sowing and 20°C-25°C during harvest and rainfall

between 50 cm to 100 cms. Ans.17 1. Temperature- 18°C - 32°C

2. Rainfall- 150 cm - 300 cm3. Soil- Deep fertile clayey or loamy soils.

Ans.18 (i) Transplantation : In transplantation, seedlings are first grown in nurseries and after 4 to 5 weeks when the saplings attain a height of 25 to 30 cm they are transplanted to prepared rice fields.

(ii) Broadcasting : In this method after ploughing, the seeds are scattered all over the field by hand before the onset of monsoon.

Page 54: CHAPTER INTERPRETATION OF TOPO-GRAPHICAL MAPS

P-54 G E O G R A P H Y - X

Ans.19 Rice WheatKharif crop Rabi crop18°C - 32°C; 150 to 300 cm of rain 10°C- 15°C; 50 to 100 cm of rainRequires alluvial soil with a subsoil of clay to allow water to stagnate

Loamy soil

Sown by broadcasting, dibbling, drilling or transplantation

Sown by drilling

Harvested by hand Harvesters used[ICSE Marking Scheme, 2014]

Ans.20 Three aspects of Japanese method of cultivation-(i) Use of High Yielding Variety (HYV) of seeds.(ii) Saplings are sown in the nursery and raised in the nursery beds for 4-5 weeks.(iii) Manure is extensively used to enhance the yield.

Ans.21 Importance of Pulses :(i) Pulses form an important part of the Indian diets because they are full of protein.(ii) Pulses are grown as rotation crops as they are leguminous crops that fix atmospheric nitrogen in the

soil and increase the natural fertility of the soil.(iii) Pulses are good cattle fodder too.

Ans.22 Major crop seasons are :(i) Kharif Crops : The cropping season is from July to October during the south west monsoon. Rice,

jowar, sugarcane, bajra, ragi, maize, cotton and jute are some of the important kharif crops.(ii) Rabi Season : The cropping season is from October to March. Crops like wheat, barley, rapeseed,

linseed, gram, peas, mustard, potatoes, etc. are grown as Rabi crops. Ans.23 (i) It enables to select only healthy seedlings for the plants.

(ii) Less wastage of seeds.(iii) It minimizes weed pressure by resowing.(iv) It gives higher yield.

ll

Page 55: CHAPTER INTERPRETATION OF TOPO-GRAPHICAL MAPS

P-55S O L U T I O N S

DRAMA

1CHAPTER

10 WORKSHEET-39Ans.1 One problem faced by the sugarcane cultivators in India-

(i) Sugarcane is a soil-exhausting crop and thus need good amount of fertilizers which increases the cost of production.

(ii) The location of sugar mills are far from the fields, thus, a delay of more than 24 hours results in the reduction of sucrose content in the canes. [Any one point]

Ans.2 Ratooning is a method in which during harvesting of sugarcane plant, the roots and the lower parts of the plant are left uncut to give the ratoon or the subtle crop.

It is associated with Sugarcane.

Ans.3 Oil cake is the residue of the oil seeds from which the oil has been extracted.[ICSE Marking Scheme, 2016]

Ans.4 As the mills are closer to the fields hence there is less loss of sucrose content/ use of better quality cane/larger farms, hence mechanised farming is possible/co-operative farming is practised, ensuring a better yield with better seeds, increased use of better fertilizers, better irrigation method and better crop protection measures/Frost free growing season/ tapering shape of peninsular- Sea breeze/Longer crushing season/regur soil responsible for better yield. [Any one point]

[ICSE Marking Scheme, 2015]Ans.5 Two non-edible oilseeds grown in India :

(i) Linseed : It is used in paints, varnishes, linoleum, oil cloth, printing inks. Linseed oil was often used for tempering wood (especially new bats—cricket, hockey etc).

(ii) Castor seed : Its oil is used in lubricants, soaps, inks, varnishes and linoleum. [Any one use of each]

Ans.6 (i) Ratooning(ii) Oil cake(iii) Coagulation [ICSE Marking Scheme, 2015]

Ans.7 (i) Sett Method and Ratoon Method.(ii) Sugarcane is a labour intensive crop and needs skilled labour for cutting the stalks from the ground

carefully and efficiently.(iii) So that sugarcane can be processed within 48 hours of cutting to preserve the sucrose content.

Ans.8 (i) 1. The mills are closer to the fields hence there is less loss of sucrose content.2. Use of better quality cane and longer crushing season.3. Grown in large farms and hence mechanised farming is possible.4. Frost free growing season. [Any two points]

(ii) One advantage of ratoon cropping- This method is inexpensive as no preparation of the field is required. One disadvantage of ratoon cropping- Ratoons produce low quality crop as with successive year the canes are thinner with low sucrose

content. Ans.9 (i) Geographical conditions necessary for the cultivation of groundnuts are :

1. Temperature- 20°C to 25°C2. Rainfall- Between 50 cm-100 cm3. Black soil, sandy loams and loamy soil are ideal for the crop.

(ii) The climatic condition that adversely affects the groundnut crop are-1. Prolonged drought2. Continuous rains

AGRICULTURE III AND IV [CASH CROPS (1) AND (2)]AND II

Page 56: CHAPTER INTERPRETATION OF TOPO-GRAPHICAL MAPS

P-56 G E O G R A P H Y - X

3. Stagnant water4. Frost

(iii) Linseed and Castor. Ans.10 Gujarat is the largest producer of Castor seed.

Important use of its oil : Used in paints, varnishes, printing ink, etc. Important use of oil cake : Used as animal fodder.

Ans.11 Problems of Sugarcane Cultivation :(i) Sugarcane is a soil-exhausting crop and thus need good amount of fertilizers which increases the cost

of production.(ii) In India the yield per hectare is extremely low as compared to other countries of the world.(iii) Sugarcane has a short crushing season normally from 4 to 7 months in a year which results in financial

problems for the industry as the mills and the workers remain idle.(iv) The location of sugar mills are far from the fields, thus, a delay of more than 24 hours results in the

reduction of sucrose content in the canes.(v) Sugarcane is an annual crop but the land available for sugarcane is less as compared to other crops,

thus, the farmers are unable to cultivate any other crop.(vi) The production cost of sugarcane in India is the highest in the world due to uneconomic process of

production, inefficient technology and heavy excise duty.(vii) Small and uneconomic size of mills.(viii) Old and obsolete machinery are used in most of the Indian sugar mills and needs rehabilitation.(ix) Sugar industry is facing competition with gur and khandsari since Khandsari industry is free from excise

duty and can offer higher prices of cane to the cane growers.(x) Sugarcane cultivation needs good amount of water but lacks irrigation facilities.(xi) The government has fixed prices for the sugarcane farmers which is not profitable for them.

[Any two points] Role of Government in Solving Farmer’s Problems-

(i) The government has set up a number of Cooperative Societies.(ii) To develop various means of irrigation to provide regular supply of water to the sugarcane fields. (iii) To provide adequate and timely loans to farmers on easy terms so that they can buy farm machinery

and other agricultural items.(iv) To educate farmers with latest farming techniques and help farmers through specially developed

programmes on radio and television. [Any two] ll

WORKSHEET-40

Ans.1 Any one of the following: Oil of the oil seeds is useful, gives edible oil and raw material for products like paints, varnishes, soaps etc. Provides employment The oil cake used for fodder Exported and so earns foreign exchange. [ICSE Marking Scheme, 2016]

Ans.2 (i) After harvesting of sugarcane plant, the roots and the lower parts of the plant are left uncut to give the ratoon or the subtle crop.

(ii) The successive crops that grow from the left out subtle is called the Ratoon. Ans.3 Oilcake is used as animal fodder and is also used as good manure in the farms. Ans.4 Gujarat is an important producer of groundnuts. Ans.5 1. Temperature- Grows best in areas with temperature between 20°C and 24°C.

2. It requires 100 cm-150 cm of rainfall throughout the year. 3. The crop grows well in well drained rich alluvial, heavy loams or lava soil.

Ans.6 Castor seed is a non-edible oil seed. Its oil is used in lubricants, soaps, inks, varnishes and linoleum.

Page 57: CHAPTER INTERPRETATION OF TOPO-GRAPHICAL MAPS

P-57S O L U T I O N S

Ans.7 (i) Due to climatic fluctuations and low market value, the farmers prefer other commercial crops.(ii) Lack of high yielding varieties, irrigation facilities and chemical fertilizers.

Ans.8 (i) Groundnut is mainly used for the manufacture of hydrogenated oil and is used in making margarine, soap, medicines, cooking oil, etc.

(ii) It is eaten raw, roasted and salted.(iii) Its oilcake is used as cattle fodder.(iv) Being a leguminous plant it plays an important role in crop rotation. [Any 3 points]

Ans.9 Linseed is obtained from the flax plant. Commercial Uses of its Oil-

(i) It is used in paints, varnishes, linoleum, oil cloth, printing inks.(ii) It is often used for tempering wood (especially new bats—cricket, hockey etc).(iii) Oil cake is used as cattle fodder.

ll

WORKSHEET-41

Ans.1 To allow excess water to drain off as tea cannot tolerate stagnant water. [ICSE Marking Scheme, 2015]

Ans.2 It means to reduce the moisture content in the tea leaves, make the leaves soft and to allow the flavour compounds to develop.

Ans.3 Tea bushes are pruned to maintain the height of the bush to facilitate the plucking of leaves and to have new shoots bearing plenty of soft leaves.

Ans.4 New alluvium fertile soil in the Ganga delta region is most suitable for jute cultivation. Ans.5 Ginning is a process of separation of cotton fibre from the cotton seed. Ans.6 (i) Karnataka, Kerala and Tamil Nadu.

(ii) Temperature- Between 15°C and 28°C. Rainfall- Between 150 cm and 200 cm of annual rainfall. Stagnant water is harmful. Soil- Well drained, rich friable loams containing a good deal of humus and minerals like iron and

calcium are ideal for coffee cultivation. Ans.7 (i) Pruning is an essential part of tea cultivation as it helps in maintaining the proper shape of tea bush

to a height of about one metre to facilitate the plucking of leaves.(ii) The coffee beans are roasted at temperatures of about 99°C and then ground into coffee powder.

Roasting gives the brown colour, aroma and taste.

Ans.8 Cotton Jute

Temp : 20°C - 30°CRain : 50 cm- 120 cm

Temp : 21°C- 35°CRain : 150 cm - 250 cm

[ICSE Marking Scheme, 2015]

Ans.9 (i) 21°C to 30°C; 50 to 75 cm of rain; black soil.

(ii) The new tea plant has all the qualities of the mother plant from which the cutting is taken and sown. [ICSE Marking Scheme, 2014]

Ans.10 (i) Retting : It is a microbiological process which loosens the outer bark and facilitates removal of the fibre from the stalk.

(ii) Ginning : It is a process of separation of cotton fibre from the cotton seed. Ans.11 Uttar Pradesh produces short staple cotton.

Temperature- 21°C and 30°C. Rainfall- 60 cm to 120 cm. Alluvial soil.

Ans.12 Gujarat leads in cotton cultivation. Two climatic factors which affect the cotton cultivation adversely are-

1. Cotton plant is extremely sensitive to frost.

Page 58: CHAPTER INTERPRETATION OF TOPO-GRAPHICAL MAPS

P-58 G E O G R A P H Y - X

2. Rainfall during the cotton ball opening and harvesting periods is harmful for the plants. Ans.13 West Bengal is the leading producer of Jute.

Rainfall- 150 cm -200 cm. Soil- New Alluvial fertile soil in the Ganga-Brahmaputra delta.

Ans.14 (i) Ginning : It is a process of separation of cotton fibre from the cotton seed.(ii) Arabica : An evergreen shrub or tree yielding seeds that produce a high quality coffee.

Ans.15 Temperature- 24°C to 30°C. Rainfall- 150 cm- 500 cm. Soil- Well drained, deep friable loamy soil.

Ans.16 (i) Temperature- 21°C and 30°C with atleast 200 frost free days.(ii) Rainfall- 60 cm to 120 cm.(iii) Soil- Deep black clay soil.

Ans.17 (i) Assam / West Bengal / Kerala / Tamil Nadu(ii) Mostly women harvest the tea because it involves fine plucking, (two leaves and a bud) and women

labourer can patiently pluck the same.(iii) Two geographical conditions are: Temperature – 24°C to 30°C Rain – 150 cm to 350 cm Soil – well drained deep friable loams, rich in organic matter. Laterite soil [Any two]

[ICSE Marking Scheme, 2016]Ans.18 (i) Jute has to be retted to facilitate the removal of the fibre from the stalk.

(ii) Tea is grown on hill slopes because water logging at the roots of the plant is injurious and so it prevents from water stagnation.

(iii) The coffee plant cannot stand direct sunrays and is thus grown under shady trees such as silver oak and banana trees.

Ans.19 (i) Coffee Arabica and Coffee Robusta.(ii) The coffee plant cannot stand direct sunrays and is thus inter-planted with orange trees, cardamom

and pepper vines.(iii) Karnataka.

Ans.20 (i) The two main varieties of coffee are-1. Arabica2. Robusta

(ii) 1. Dalap and Silver Oak are grown in coffee estates as shade trees as they protect the coffee plants from the direct rays of the sun.

2. Orange and Plum are grown to provide extra income to the farmers. Ans.21 Tea is considered a labour intensive crop because most of the tea cultivation activities are done

manually by the skilled and unskilled farmers and labourers like sowing, pruning, plucking, etc. Two states in India where tea is widely grown are : Assam and West Bengal.

ll

Page 59: CHAPTER INTERPRETATION OF TOPO-GRAPHICAL MAPS

P-59S O L U T I O N S

DRAMA

1CHAPTER

11 WORKSHEET-42Ans.1 On the basis of the nature of products, the industries are classified into :

(i) Heavy Industries : Ship building, machinery manufacturing, iron and steel.(ii) Light Industries : Sewing Machines, cycles, electronic goods.

Ans.2

Public Sector Industry Private Sector Industry

1. These industries are owned and managed by Central Government or State Government which includes industries of public utility.

1. These industries are owned and managed privately by individuals or group of individuals.

2. E.g. Post and telegraph, railways, oil refineries, heavy engineering industries, defence establishments, Bharat Heavy Electricals Limited (BHEL), Steel Authority of India Limited (SAIL), Gas Authority of India Limited (GAIL), etc.

2. E.g. Industries like Reliance India Limited, Wipro, Infosys, TCS, etc.

Ans.3 (i) Agro-based industries provide agricultural raw materials to the industries.(ii) These industries are good foreign exchange earner.

Ans.4

BASIC INDUSTRIES CONSUMER INDUSTRIES

1. These Industries depend on other industries for their manufacturing.

1. These industries process the basic raw materials into primary goods for direct use by the consumers.

2. Iron and steel industry and petroleum industry are major basic industries.

2. Textile industry, Sugar Industry, Automobile industry are some main consumer industries.

Ans.5 (i) It provides employment to a large number of people.(ii) It is a subsidiary occupation since agriculture alone cannot support livelihood to all rural people.

Ans.6 (i) Industry should be set up close to the availability of the raw material.(ii) Nearness to the market.(iii) Proper transportation infrastructure for transporting the raw materials to the industry.(iv) Cheap and regular supply of power.

Ans.7 (i) India is an agricultural country and through industrialisation the development of agriculture can be initiated.

(ii) Only agriculture cannot generate employment and thus establishment of industries can generate employment opportunities on a large scale.

(iii) The development of industries producing capital goods i.e. machines, equipment etc. enables a country to produce a variety of goods in large quantities and at low costs and make for technological progress.

(iv) Through industrialisation infrastructures like railways, roadways, dams, etc. can be constructed which can enhance the future growth of Indian economy.

(v) Industrialisation is a necessity for country’s security because only through industrial development self-reliance in defence can be achieved where she can produce her own defence materials.

(vi) Expansion of industries in the backward regions of India is needed to counter the regional imbalance. [Any three points]

AGRO-BASED INDUSTRY

Page 60: CHAPTER INTERPRETATION OF TOPO-GRAPHICAL MAPS

P-60 G E O G R A P H Y - X

Ans.8 Cottage industries are organised by individuals with private resources and with the help of their family members and their skills e.g. weaving, handloom, carpet industry, etc.

Cottage industries are important because :(i) It provides employment to a large number of people.(ii) It is a subsidiary occupation since agriculture alone cannot support livelihood to all rural people.

ll

WORKSHEET-43Ans.1 Cotton textile industry is called an agro-based industry because it requires cotton as its raw material

which is an agricultural product. Ans.2 Karnataka. Ans.3 (i) Competition from artificial silk which is cheap and of better quality.

(ii) Import of cheap and alternative textiles from China and other Asian countries.(iii) Use of outdated manufacturing technology, primitive and unscientific ‘reeling’ and ‘weaving’

techniques, etc.(iv) The price fluctuation of raw silk affects the weavers and the industry.(v) Lack of new technologies and modern power looms is affecting the growth of production.

[Any One] Ans.4 (i) Regular supply or proximity to raw material.

(ii) Favourable climatic conditions.(iii) Good network of road and rail transportation within the country and sea routes for the international

market.(iv) Location of major ports facilitates the export and import facilities.(v) Availability of cheap and skilled labours. [Any two points]

Ans.5

Mineral-based Industry Agro-based Industry

1. These industries use minerals both metallic and non-metallic as raw materials.

1. These industries depend on the raw materials produced by the agricultural sector.

2. These industries produce both consumer and value based goods.

2. They mostly produce consumer goods.

3. These industries supply their raw material to iron and steel industry, heavy engineering and machine tool industry, cement industry, etc.

3. The major Agro-based industries are cotton, sugar, jute, tea, coffee, etc.

Ans.6 (i) The maritime climate of Maharashtra which is free from loo and frost.(ii) The availability of black soil which is well drained and more fertile than alluvial soil.(iii) Excellent transport facilities in Maharashtra has given it an advantageous position in relation to export

markets.(iv) The sugar factories are located close to the sugarcane farms which prevent the loss of sucrose content

due to minimum transportation time.(v) The farmers have new machinery and crushing devices which ensures high yield.

[Any two points]

Ans.7 Fluctuation in availability of raw material as it is agricultural based / outdated machines / frequent power cut. [ICSE Marking Scheme, 2016]

Ans.8 Availability of raw material as it is a jute producing area / Plenty of clean water for retting / availability of labour and transport facility. [Any two]

[ICSE Marking Scheme, 2016]

ll

Page 61: CHAPTER INTERPRETATION OF TOPO-GRAPHICAL MAPS

P-61S O L U T I O N S

WORKSHEET-44

Ans.1 (i) As it requires cotton, an agricultural product as its raw material.(ii) It is more widespread than the jute textile industry because cotton is grown all over the country/

whereas jute is cultivated mainly in the east of India/ there continues to be a great demand for cotton all over the country/hence mills are set up everywhere; whereas the demand for jute is declining as it is facing stiff competition from synthetic material like nylon and plastic/ India being a tropical country more demand for cotton fabrics/ affordable by mass. [Any one point]

[ICSE Marking Scheme, 2015]

Ans.2 (i) One important point of similarity between the woollen and silk industry is that both use animal products as raw materials/ both are small-scale/cottage industries. [Any one point]

(ii) Woollen industry- Punjab; Silk Industry- Karnataka [ICSE Marking Scheme, 2015]

Ans.3 Wool - Jammu & Kashmir / Punjab / Haryana / Himachal Pradesh / Uttar Pradesh Silk - Karnataka / Andhra Pradesh / Tamil Nadu / West Bengal / Bihar / Jharkhand / Assam.

[ICSE Marking Scheme, 2014]

Ans.4 Any two of the following:(i) Water from the Ganga and her distributaries Power from DVC Kolkata is a port Well connected by road and rail Labour is easily available from West Bengal, Odisha, Jharkhand.(ii) India is a tropical country, so the demand for wool is not as much as for cotton. Woollen goods are not as profitable as cotton, so more investment in the cotton industry. Woollen

goods are not as affordable as cotton The sheep do not have a thick or good quality fleece Competition from synthetic material. [ICSE Marking Scheme, 2014]

Ans.5 (i) The rearing of silk worms for the production of silk is known as Sericulture.(ii) Karnataka is famous for silk industry because-

1. It has favourable climate for rearing silkworms.2. The state-owned Channapatna has a capacity of numerous spindles.3. There are nurseries, silk farms and licensed seed distributors.4. The decentralized sector has many powerlooms and handlooms. [Any two points]

Ans.6 (i) Tasar Silk(ii) Bagasse(iii) Sericulture

Ans.7 (i) (a) Regular supply or proximity to raw material.

(b) Favourable climatic conditions specially the humid climate.

(c) Good network of road and rail transportation within the country and sea routes for the international market.

(d) Location of major ports facilitates the export and import facilities.

(e) Availability of cheap and skilled labours.

(f) Mumbai is a centre of financial and commercial resources.

(g) Electricity is supplied by the Tata Hydroelectricity system in the Western Ghats to Mumbai. [Any two points]

(ii) Ahmedabad and Coimbatore.

ll

Page 62: CHAPTER INTERPRETATION OF TOPO-GRAPHICAL MAPS

P-62 G E O G R A P H Y - X

WORKSHEET-45

Ans.1 (i) Climatic condition of Karnataka is suitable for mulberry tree and so sericulture can be practiced.

(ii) Tassar, Eri, Muga [Any two](iii) Uttar Pradesh – Mirzapur / Pratapgarh / Shahjahanpur [Any one] Tamil Nadu – Coimbatore / Salem / Tanjavur / Tirunelveli [Any one]

[ICSE Marking Scheme, 2016]

Ans.2 (i) India is a tropical country and so demand of woollen is low / Poor quality of indigenous wool / Woollens are expensive so less demand / Winter is severe only in north so there is small market.

(ii) Amritsar, Ludhiana, Dhariwal. [ICSE Marking Scheme, 2016]

Ans.3 (i) Problems of the sugar industry: poor quality of cane/out-dated machinery/fluctuating supply of raw material/low profit margins as prices are fixed by the govt./inadequate use of by-products, increasing production costs/mills are located far from fields, leading to loss of sucrose/ the seasonal nature of the industry, increasing production costs/small size of farms/inadequate use of fertilizers.

[Any two problems]

(ii) By- products- bagasse, molasses, pressmud. [Any two points][ICSE Marking Scheme, 2015]

Ans.4 (i) Introduction of artificial silk which is more durable / Easy to maintain / crease proof / cheaper to produce /Need for modern power looms for increase in production/ No systematic testing and grading of silk/ changes in price of raw silk affect both growers and the industry. [Any one points]

(ii) Two products of the jute industry- carpets/ wall- hangings/pot holders fabrics/sandals/hand-bags. [Any two points]

(iii) As they are cheaper/are more durable /are moth resistant/ not dependent on agriculture for raw materials / Chemicals are easily available. [Any one points]

[ICSE Marking Scheme, 2015]

Ans.5 Any three of the following : Agro-based industry, therefore the supply fluctuates Unproductive workers Outdated machinery and methods of processing jute Competition from synthetic materials Competition from countries like Bangladesh. [ICSE Marking Scheme, 2014]

Ans.6 Any three of the following : Sugarcane grows better in the south because it is a tropical crop and because of the black soil and

more scientific methods of cultivation. The sugar mills are near the plantations, so there is no loss of sucrose. This also lowers the transport costs. The sugar industry is better managed in the south. The factories are near the centres of consumption. [ICSE Marking Scheme, 2014] Ans.7 1. The cotton textile industry contributes nearly 30% of the value of exports and employs more than

55 million people.

2. The industry is one of the largest foreign exchange earner commodities of India.

3. It contributes about 14% to the industrial production, 4% to the GDP and 14.42% to the country’s export earnings.

Ans.8 (i) Maharashtra and Gujarat.(ii) (a) Regular supply or proximity to raw material.

(b) Favourable climatic conditions specially the humid climate.(c) Good network of road and rail transportation within the country and sea routes for the

international market.

Page 63: CHAPTER INTERPRETATION OF TOPO-GRAPHICAL MAPS

P-63S O L U T I O N S

(d) Location of major ports facilitates the export and import facilities.(e) Availability of cheap and skilled labours. [Any two points]

Ans.9 (i) Two reasons for the importance of the cotton textile industry-

1. The cotton textile industry contributes nearly 30% of the value of exports and employs more than 55 million people.

2. The industry is one of the largest foreign exchange earner commodities of India.

(ii) One reason for its poor performance-

1. Many factories are old, obsolete and sick industrial units and thus faces low productivity. ll

WORKSHEET-46Ans.1 Bangalore. Ans.2 Cotton grows widely in Maharashtra due to ideal climatic conditions, i.e. 21°C to 30°C of temperature,

50 to 75 cm of rain and black soil.

Ans.3 Uttar Pradesh in North India and Tamil Nadu in South India.

Ans.4 (i) Cotton in Ahmedabad

(ii) Silk in Mysuru

Ans.5 (i) It is so because after harvesting the sucrose content in the cane begins to decrease.

(ii) Favourable climatic conditions with temperatures between 16°C and 32°C which is ideal for rearing the silkworms.

Ans.6 Molasses- It is thick, dark brown juice obtained from raw sugar during the refining process.

Bagasse- It is the dry pulpy residue left after the extraction of juice from sugarcane.

Ans.7 The rearing of silk worms for the production of silk is known as Sericulture.

The two types of silk are- Mulberry and Tasar.

Ans.8 The silk industry considered as a small scale industry because-

(a) Low capital investment i.e. less than one crore.

(b) It is a farm based and labour intensive industry and requires less machinery.

Two types of silk produced in India- Mulberry and Tasar.

Ans.9 Bengaluru (Bengalore) and Mysuru (Mysore).

Ans.10 (i) The Indian silk industry is an integral part of Indian Textile Industry and is one of the largest producers of silk in the world.

(ii) The silk industry in India employs 60 lakh workers.

Ans.11 Two reasons for the concentration of the sugar industry in Uttar Pradesh-

(i) Availability of fertile alluvial soil in the Ganga-Yamuna doab area.

(ii) Provides cheap labour as it is densely populated.

Ans.12 (i) Handloom industry is one of the oldest industries of India providing employment to millions of skilled and unskilled people.

(ii) The handloom industry is mainly located in small town and rural areas and thus preserve our heritage and culture.

(iii) They earn valuable foreign exchange. [Any two points]

Ans.13 Two economic advantages of the Handloom Industry-

(i) It provides employment.

(ii) It earns foreign exchange.

Ans.14 (i) Kolkata has many cotton mills though cotton is not grown in West Bengal because :

(a) Kolkata is located very close to coal mines.

(b) It has abundant capital supply.

Page 64: CHAPTER INTERPRETATION OF TOPO-GRAPHICAL MAPS

P-64 G E O G R A P H Y - X

(c) There is availability of cheap labour, humid climate, plenty of soft water.

(d) It has port facility and excellent means of transport and communication. [Any one point]

(ii) (a) Karnataka has favourable climatic conditions with temperature ranging from 16°C to 32°C.

(b) Sericulture provides employment to a large number of people. [Any one point]

(iii) (a) The sugarcane produced in India is of poor quality with low sucrose content.

(b) In rural areas, instead of sugar, gur and khandsari are in more demand.

Ans.15 Two by-products of Sugarcane–

Molasses : It is used in the production of citric acid, chemicals, synthetic rubber and as fuel for mills.

Bagasse : It is used as a biofuel and in the manufacture of pulp and building materials.

Pressmud : It is used for making wax, carbon paper and shoe polish.

Ans.16 (i) Regular supply or proximity to raw material.

(ii) Favourable climatic conditions specially the humid climate.

(iii) Good network of road and rail transportation within the country and sea routes for the international market.

(iv) Availability of cheap and skilled labours.

Ans.17 (a) Kolkata is located very close to coal mines.

(b) It has abundant capital supply.

(c) There is availability of cheap labour, humid climate, plenty of soft water.

(d) It has port facility and excellent means of transport and communication. [Any three points]

Ans.18 (i) It is so because after harvesting the sucrose content in the cane begins to decrease.

(ii) Four sugar milling centres in the northern plains are Saharanpur, Meerut, Muzaffarnagar and Baghpat.

ll

Page 65: CHAPTER INTERPRETATION OF TOPO-GRAPHICAL MAPS

P-65S O L U T I O N S

DRAMA

1CHAPTER

12 WORKSHEET-47

Ans.1 Vishakhapatnam has an important ship-building yard because Vishakhapatnam is a sea port, has the facility of a dry dock, gets its iron and steel from VISL, power from the Nagarjunasagar Dam, labour from Andhra Pradesh, Odisha (Orrisa) and Karnataka. [ICSE Marking Scheme, 2014]

Ans.2 Mini steel plants uses electric furnaces while integrated steel plants use blast furnaces. Ans.3 (i) German Firm, Krupps and Demag.

(ii) Iron ore from the reserves of Sundargarh and Keonjhar district of Odisha. Coal is obtained from Jharia, Talcher and Korba fields.

Ans.4 (i) Tata Iron and Steel Plant (TISCO).(ii) Mini Steel plants are smaller units, work through electric furnaces and mainly uses steel scrap and

sponge iron or pig iron as their raw material. Mini steel plants have the following advantages-

(a) Scarp iron is used as raw material which is cheap and easily accessible.(b) These plants can be built with less capital investment.(c) It does not cause pollution since it runs on electric power. [Any one point]

Ans.5 ‘The Iron and Steel industry constitutes the backbone of modern industrial economy’ due to the following reasons-

(i) It provides raw material to industries for manufacturing industrial machinery, railway engines, railway tracks, electrical machinery, defence equipments, bridges, dams, shops, automobiles, etc.

(ii) India has one of the richest reserves of all the raw materials required for the industry, namely, capital, cheap labour, iron ore, power, coal, etc.

Ans.6 (i) Rourkela Iron and Steel Plant.(ii) Iron ore from the reserves of Sundargarh and Keonjhar district of Odisha. Manganese is obtained

from Barajmda.

Ans.7 It uses scrap iron / uses electric furnace so no pollution / conserves coal / help in decentralized development. [ICSE Marking Scheme, 2016]

Ans.8 The iron and steel plant at Durgapur was established with British collaboration. Iron ore – Singbhum in Jharkhand / Keonjhar in Odisha Manganese – Keonjhar in Odisha Coal – Jharia coalfield in Jharkhand / Raniganj in West Bengal [ICSE Marking Scheme, 2016]

Ans.9 (i) Iron and Steel(ii) Electric locomotives(iii) Aero-engines [ICSE Marking Scheme, 2016]

Ans.10 (i) Engines for MIG aircraft - Koraput in Odisha (Orissa)(ii) Diesel locomotives - Varanasi in Uttar Pradesh(iii) Software - Bengaluru (Bangalore)/ Hyderabad / Pune. [ICSE Marking Scheme, 2014]

Ans.11 (i) 1. It does not cause pollution and the cost of production is low. 2. Mini steel plant uses scrap iron, sponge iron, etc. as raw material from the large steel plants.(ii) Iron Ore from Dalli Rajhara mines Coal is obtained from Bokaro, Kargati and Jharia fields in Jharkhand and Korba in Chhattisgarh.

ll

WORKSHEET-48Ans.1 Bhilai Steel Plant.

MINERAL RESOURCES

Page 66: CHAPTER INTERPRETATION OF TOPO-GRAPHICAL MAPS

P-66 G E O G R A P H Y - X

Ans.2 Rourkela Steel Plant. Ans.3 Germany.

Ans.4 (i) It is called a basic industry because it supplies raw materials to all other industries. / Key product for making of all industries/ used for automobiles/ railways / shipbuilding / heavy machineries / tools / bridges/ roads/ domestic purpose etc

(ii) A mini steel plant is one which manufactures steel using scrap iron in electric arc furnaces.[ICSE Marking Scheme, 2015]

Ans.5 (i) Kanpur- aircraft/sugar/jute/electronic goods/automobiles(scooters) Leather industries.[Any one point]

(ii) Rourkela- iron and steel/fertilizers. [Any one point](iii) Pune- automobiles/ electronics/ cotton textile. [Any one point](iv) Mangalore- oil refining [ICSE Marking Scheme, 2015]

Ans.6 Tata Iron and Steel Company Iron - Singhbhum in Jharkhand / Mayurbhanj in Odisha (Orissa) / Bonai in Odisha (Orissa) Coal - Jharia in Jharkhand / Bokaro in Jharkhand. [ICSE Marking Scheme, 2014]

Ans.7 (a) Iron Ore from Dalli Rajhara mines(b) Coal is obtained from Bokaro, Kargati and Jharia fields in Jharkhand and Korba in Chhattisgarh.(c) Limestone developed in Nandini near Bhilai.(d) Manganese obtained from neighbouring district of Balaghat.

Ans.8 (i) Availability of iron, coal, manganese, limestone.(ii) Supply of cheap labour from the states of Bihar and West Bengal.

Ans.9 (i) It is so because it lacks the basic raw material required for iron and steel industry.(ii) Iron and Steel industries require large land area, huge capital investments, modern technical

knowledge, huge machineries, etc. Ans.10 (i) 1. Mini Steel plants are smaller units than Iron and steel Industry.

2. It works through electric furnaces.3. It mainly uses steel scrap and sponge iron or pig iron as their raw material.

(ii) The integrated steel plant at Jamshedpur gets its- Iron ore from Gurumahisani mines in Mayurbhanj district of Odisha and Noamundi mines in Singhbhum district of Jharkhand. Coal from the Jharia and Bokaro coalfields.

Ans.11 (i) A unified steel mill where all the primary functions of producing steel is carried out like iron making, steel making, casting, roughing rolling and product rolling.

(ii) Bhilai Steel Plant. Iron Ore from Dalli Rajhara mines Coal is obtained from Bokaro, Kargati and Jharia fields in Jharkhand and Korba in Chhattisgarh.

ll

WORKSHEET-49Ans.1 Bengaluru and Hyderabad. Ans.2 Pune- Electronics Industry.

Ans.3 Organic chemicals like naphtha, ethylene and benzene. The raw materials are easily available since they are not agro-based Replacing traditional raw

materials like wood and metal Cheap Durable. [ICSE Marking Scheme, 2014] Ans.4 (i) (a) India has been experiencing a strong growth in the demand of consumer products and durables

in recent years which have facilitated the growth in the electronics, especially in the telecom sector both directly and indirectly.

Page 67: CHAPTER INTERPRETATION OF TOPO-GRAPHICAL MAPS

P-67S O L U T I O N S

(b) Indian electronic industry plays an important role in space technology and has launched many indigenous satellites like APPLE and INSAT-1 and INSAT -2 Series.

(ii) Bengaluru and Hyderabad. Ans.5 Chemical obtained either directly or indirectly from chemical processing, of petroleum oil or natural

gas is called Petrochemicals. Two products made from petrochemicals are dyes, synthetic fibre, synthetic rubber, detergents,

insecticides and pesticides. [Any two] Ans.6 Two geographical reasons for the growth of IT industries in Bangalore-

(i) Cool and moderate climate neither cold nor warm.(ii) Abundant supply of electricity through the hydropower projects.(iii) Bengaluru (Bangalore) is well connected to other major metropolises of Madras and Hyderabad.

[Any two] Ans.7 Bengaluru (Bangalore) is known as the electronic capital of India.

Two major centres of electronic products- Hyderabad, Bengaluru (Bangalore), Delhi, Pune, Mumbai, etc.

Ans.8 (i) Natural gas and Crude oils are the two raw materials required by the petrochemical industry.(ii) Two products are Adhesives, fertilizers, dyes, synthetic fibre, synthetic rubber, detergents, insecticides

and pesticides. (iii) The petrochemical products are gaining importance because :

(a) Petrochemicals are cost effective, economic and cheap.(b) The raw material is easily available and not depended on agricultural raw material.

Ans.9 (i) Petrochemical Industry : Adhesives and Fertilizers.(ii) Heavy Engineering Industry : Not in Syllabus.(iii) Electronic Industry : Computers and Television.

Ans.10 (i) This is because products made from petrochemicals are cheaper and more durable(ii) A heavy engineering industry requires a huge capital investment because it requires a big plot of

land, a big labour force, consumes power and the raw material is heavy and expensive(iii) The electronic industry is a fast developing industry because electronics are used in many fields,

computers, entertainment, medicine, engineering and education which is important for the development of a country. [ICSE Marking Scheme, 2016]

ll

WORKSHEET-50

Ans.1 The electronic industry has had an effect both on entertainment because electronics are used in televisions, video players, tape recorders, audio systems Education has been affected because electronics are used computers, LCD projectors and software has been prepared to be used as teaching aids. [ICSE Marking Scheme, 2014]

Ans.2 Two petrochemical units are-1. Herdillia Chemicals Ltd. in Chennai.2. National Organic Chemicals Industries Ltd. near Mumbai.3. Petrofils Cooperative Limited (PCL) in Vadodara.4. Indian Petrochemical Corporation Ltd. near Vadodara. [Any two units]

Ans.3 Most of the petrochemical units are in Maharashtra and Gujarat because-(i) Maharashtra and Gujarat have oil refineries which can supply the basic raw materials of Naphtha or

Ethylene and Benzene to the petrochemical industry.(ii) Both the states have excellent port facilities for import and export.

Ans.4 Petro-chemical are those chemicals obtained either directly or indirectly from chemical processing, of petroleum oil or natural gas e.g. dyes, synthetic fibre, synthetic rubber, detergents, insecticides and pesticides.

Petrochemicals are 1. cost effective2. economic and cheap.

Page 68: CHAPTER INTERPRETATION OF TOPO-GRAPHICAL MAPS

P-68 G E O G R A P H Y - X

Ans.5 Four petro-chemical products-(i) Synthetic fibre(ii) Synthetic rubber(iii) Detergents(iv) Insecticides.

Ans.6 (i) In defence electronics are used for making remote weapon, telecommunication, long distance transmissions, etc.

(ii) Cool and moderate climate neither cold nor warm. Ans.7 (a) Space Technology : Indian electronic industry plays an important role in space technology and has

launched many indigenous satellites like APPLE and INSAT-1 and INSAT -2 Series.(b) Entertainment : Electronics are used in televisions and video players, computers and LCD

projectors. Ans.8 Two centres- Vadodara and Chennai.

Two products- Adhesives and fertilizers. Ans.9 (i) Agriculture : Chemical industry provides fertilizers.

(ii) Public Health : Chemicals also produce medicines and vaccines for the improvement of health.

Ans.10 (i) This is because petrochemicals are cheaper/ durable, are not dependent on agricultural raw materials/ hence no fluctuation in production/ cost effective / raw materials easily available. [Any two points]

(ii) Two products are- Plastic, synthetic rubber, PVC pipes, synthetic fibre, detergents.[Any two points]

[ICSE Marking Scheme, 2015]

Ans.11 (i) Heavy engineering industries require large amounts of power, labour intensive, High capital/bulky raw materials/ High transport cost. [Any two points]

(ii) Ship building yard on the east coast- Vishakhapatnam or Kolkata; electric locomotive- Chittaranjan.[ICSE Marking Scheme, 2015]

Ans.12 (i) Not in Syllabus(ii) Bengaluru(iii) Haldia Petrochemical Ltd. in West Bengal.

ll

Page 69: CHAPTER INTERPRETATION OF TOPO-GRAPHICAL MAPS

P-69S O L U T I O N S

DRAMA

1CHAPTER

13 WORKSHEET-51

Ans.1 Advantage of the GQ project-Connects major cities and ports/provides an impetus to truck transport/enables the industrial growth of the small towns through which it passes/provides opportunities for the transport of agricultural produce from the hinterland to major cities and ports.

[Any one point] [ICSE Marking Scheme, 2015]

Ans.2 (i) Roadways are ideal for short distances as through it every village and hamlet can be reached.(ii) It transports people and goods quickly and easily.(iii) It provides door-to-door services. [Any two points]

Ans.3 Railways can carry more tonnage and passengers than airways/facilitates easier movement of bulky goods/helped to transfer new ideas/innovations to rural areas/they provide a more comfortable journey especially for elders and infants/ Inexpensive/ contributes to the economic growth and urbanisation. [Any two points]

[ICSE Marking Scheme, 2015]

Ans.4 Any two reasons:– It is more suitable for short distance travel– It provides door to door services– Perishable products can be transported easily.– Saves on cost of transporting from the station. [ICSE Marking Scheme, 2014]

Ans.5 (i) Rail transport is cheap and helps in reaching the far and inaccessible areas.(ii) 1. Passengers are travelling without tickets and sitting on the roof of the trains.

2. Rise in rail accidents.3. The railway requires replacing the old rail tracks.

ll

WORKSHEET-52Ans.1 The North-South Corridor would connect Srinagar in Jammu and Kashmir with Kanyakumari in

Tamil Nadu.

Ans.2 (i) There are no traffic lights on expressways, unlike on highways/ two- wheelers and three-wheelers are not permitted on expressways / there are no speed-breakers on expressways. [Any two points]

(ii) The first Expressway-Ahmedabad-Vadodara Expressway [ICSE Marking Scheme, 2015] Ans.3 Road is favoured in the northern plains because-

(i) It is a region of flat level land.(ii) The cost for constructing rail tracks is low.(iii) The northern region is a densely populated area.(iv) The northern plains are rich in agricultural resources. [Any two points]

Ans.4 (i) Railway lines cannot be constructed everywhere specially in the hilly regions and remote dense forested regions.

(ii) Railway transport is unsuitable and uneconomical for short distance and small traffic of goods. Ans.5 Golden Quadrilateral is the largest Express Highway project in India that connects India’s four biggest

metropolitan cities-Delhi, Mumbai, Kolkata and Chennai. Two ways in which it will help in the economic development of the country-

(i) Establishing faster transport networks between major cities and ports.(ii) Providing fast and smooth movement of products and people within India.(iii) Developing industries and creating job opportunities in smaller towns through access to markets.

ll

TRANSPORT

Page 70: CHAPTER INTERPRETATION OF TOPO-GRAPHICAL MAPS

P-70 G E O G R A P H Y - X

WORKSHEET-53

Ans.1 North rivers flow over flat terrain, whereas Deccan rivers flow over uneven land, hence presence of cataracts and rapids which hinder navigation/northern rivers are perennial, whereas Deccan rivers are seasonal. [Any one point]

[ICSE Marking Scheme, 2015]Ans.2 (i) Inland Waterway is declining because water transport is much slower than its roads, rail or air

transport.(ii) Area of water transport is restricted. It provide access to limited areas.

Ans.3 (i) Air transport is expensive.(ii) It connects only major cities.(iii) It is dependent on weather conditions and can get delayed or cancelled causing inconvenience to

passengers. [Any one point] It is still popular means of transportation because-

(i) Air transport is the fastest and most comfortable mode of transport.(ii) It can easily reach to remote and inaccessible areas like mountains, forests, deserts etc.

Ans.4 Helicopter provides services to the hilly regions, small islands, regions where rivers are flowing, dense forest and remote areas.

Ans.5 Any one of the following: Advantage : Cheapest means of transport Suitable for heavy and bulky goods Fuel efficient Eco-friendly No traffic congestion Comfortable with space for passengers Any one of the following: Disadvantage: Depends on weather conditions Slow means of transport Can saw sea sickness Restricted means of transport, can only be possible where rivers have to be navigable.

[ICSE Marking Scheme, 2014] Ans.6 (i) 1. The rivers are rain-fed and seasonal.

2. They are shorter.3. These rivers have a number of waterfalls. [Any two points]

(ii) Chennai and Vishakhapatnam. Ans.7 (i) National Waterway No.2 (NW-2)-The river Brahmaputra connects Dhubri-Pandu-Tezpur-Neamati-

Dibrugarh-Sadiya and connects the North East region with Kolkata and Haldia Ports.(ii) Haldia.

ll

Page 71: CHAPTER INTERPRETATION OF TOPO-GRAPHICAL MAPS

P-71S O L U T I O N S

DRAMA

1CHAPTER

14 WORKSHEET-54Ans.1 It is the process by which a body of water becomes enriched in dissolved nutrients that stimulate the

growth of aquatic plant life usually resulting in the depletion of dissolved oxygen.

Ans.2 Becomes an eyesore for spoils the landscape Causes pollution Affects terrestrial life adversely Leads to health hazards. [ICSE Marking Scheme, 2014]

Ans.3 (i) Due to Methyl Isocynate Gas(ii) Due to Methyl Mercury(iii) Due to Nuclear Radiation.

ll

WORKSHEET-55Ans.1 Sewage is a type of wastewater either domestic or municipal wastewater that is produced from a

community of people e.g. wastewater from sinks, cloth washers, water used to flush toilets, soap water, etc.

Ans.2 Waste management is a important because-(i) Improperly stored refuse can cause health, safety and economic problems.(ii) Transmission of diseases due to accumulation of wastes is a major threat to people and

environment. Ans.3 The three sources of Methane gas are-

(i) Mining(ii) Wetlands(iii) Landfills

Ans.4 Acid rain is the result of excessive acids in rainwater which are formed when oxides like nitrogen dioxide and sulphur dioxide react with water.

Any two of the following: Increases acidity in the soil and destroys forests and crops Corrodes buildings, monuments, bridges Contaminates air and water Affects the nervous system and health in general Aquatic life affected adversely. [ICSE Marking Scheme, 2014]

ll

WORKSHEET-56Ans.1 One source of gaseous waste is automobiles, factories, burning of fossil fuels. [Any one]Ans.2 The main objective is to minimise the generation of gaseous waste. Ans.3 Asthma and Bronchitis.

Ans. 4 (i) Domestic waste Industrial waste Agricultural waste Municipal waste [Any two]

(ii) It is a waste which decompose through the actions of bacteria, fungi and other living organisms.[ICSE Marking Scheme, 2016]

WASTE MANAGEMENT

Page 72: CHAPTER INTERPRETATION OF TOPO-GRAPHICAL MAPS

P-72 G E O G R A P H Y - X

Ans.5 (i) Waste should be segregated to make for easier waste management/ to help identify the best method of waste disposal. [Any one point]

(ii) Nuclear waste is hazardous as it can remain radio-active for thousands of years/ if it is not disposed off properly it continues to be hazardous/it can enter the human body food and water/ the damage it causes is irreparable/ it affects future generations. [Any one point]

[ICSE Marking Scheme, 2015]

Ans.6 (i) Biodegradable waste is waste that can be easily broken down by natural processes of decomposition.(ii) Exhaust from vehicles/burning of fossil fuels in factories and thermal power plants/burning of wheat

or rice straw/methane from cattle sheds. [Any one point][ICSE Marking Scheme, 2015]

Ans.7 (i) Collection of Municipal Solid Wastes- By organising house-to-house collection of solid wastes and keeping bio-medical and industrial wastes separate.

(ii) Storage of Municipal Solid Wastes- Storage facilities or bins should have easy to operate design for handling, transfer and transportation of waste.

(iii) Transportation of Municipal Solid Wastes- Covered vehicles to prevent the wastes from being scattered.

(iv) Segregation of Municipal Solid Wastes- Undertake phase programme to ensure community participation in waste segregation. [Any three]

ll

WORKSHEET-57

Ans. (i) Composting of waste is an aerobic method of decomposing solid wastes.(ii) Incineration – It is the method of burning the waste to reduce its weight and volume so that it can be

disposed easily.(iii) Segregation – It refers to separation of waste into different categories of waste. The most popular

segregation is biodegradable and non-biodegradable. [ICSE Marking Scheme, 2016]ll

WORKSHEET-58Ans.1 The process of changing the waste and non-usable materials into potentially useful materials is called

Recycling.

Ans.2 Learn to reduce consumption of goods/learn to recycle and reuse. [Any one point][ICSE Marking Scheme, 2015]

ll